Top Banner
Endocrine 05May2009 DO NOT DISTRIBUTE - 1 - Endocrine #1 – Histology 1) Which of the following coordinates most of the endocrine functions of the body? a) Anterior pituitary b) Posterior pituitary c) Hypothalamus d) Cerebral cortex e) Autocrine cells 2.1) Which of the following hormones does not dissolve in the blood and thus requires plasma proteins or specialized carrier proteins? a) Steroid hormones b) Thyroid hormones c) Small peptides, proteins, and glycoproteins d) Catecholamines e) A & B f) C & D 2.2) Where are amino acid analogs and their derivatives (including catecholamines) mainly produced? a) Ovaries, testes, and adrenal cortex b) Hypothalamus, pituitary, and pancreas c) Thyroid and parathyroid d) Neurons and adrenal medulla e) Enteroendocrine cells of the GI tract and respiratory system 3.1) Most protein hormones and catecholamines use which of the following second messenger systems for initiating the hormone-receptor interaction? a) Adenylate cyclase/camp system b) Tyrosine kinase system c) Phosphatidylinositol system d) Activation of ion channels e) None of the above (intracellular receptors) 3.2) Which of the following is NOT a second messenger? a) cAMP b) 5-HT c) DAG d) IP3 e) Ca2+ 3.3) Which of the following is NOT true of the intracellular hormone receptor system? a) Usually localized in the cell nucleus b) Is used by steroid and thyroid hormones (hydrophobic) c) Receptors contain complexes with three binding sites d) Binding causes allosteric receptor transformation that binds chromosomal DNA and activates or inhibits RNA polymerase activity e) Receptors influence gene expression but require second messengers 4.1) What portion of the pituitary gland (hypophysis) is most anterior? a) Pars distalis b) Pars intermedia c) Pars tuberalis
84

Endocrine Pre-test self test

Apr 28, 2015

Download

Documents

Thyroid, thymus, pituitary, and adrenal.
Welcome message from author
This document is posted to help you gain knowledge. Please leave a comment to let me know what you think about it! Share it to your friends and learn new things together.
Transcript
Page 1: Endocrine Pre-test self test

Endocrine 05May2009

DO NOT DISTRIBUTE - 1 -

Endocrine #1 – Histology

1) Which of the following coordinates most of the endocrine functions of the body?

a) Anterior pituitary

b) Posterior pituitary

c) Hypothalamus

d) Cerebral cortex

e) Autocrine cells

2.1) Which of the following hormones does not dissolve in the blood and thus requires

plasma proteins or specialized carrier proteins?

a) Steroid hormones

b) Thyroid hormones

c) Small peptides, proteins, and glycoproteins

d) Catecholamines

e) A & B

f) C & D

2.2) Where are amino acid analogs and their derivatives (including catecholamines)

mainly produced?

a) Ovaries, testes, and adrenal cortex

b) Hypothalamus, pituitary, and pancreas

c) Thyroid and parathyroid

d) Neurons and adrenal medulla

e) Enteroendocrine cells of the GI tract and respiratory system

3.1) Most protein hormones and catecholamines use which of the following second

messenger systems for initiating the hormone-receptor interaction?

a) Adenylate cyclase/camp system

b) Tyrosine kinase system

c) Phosphatidylinositol system

d) Activation of ion channels

e) None of the above (intracellular receptors)

3.2) Which of the following is NOT a second messenger?

a) cAMP

b) 5-HT

c) DAG

d) IP3

e) Ca2+

3.3) Which of the following is NOT true of the intracellular hormone receptor system?

a) Usually localized in the cell nucleus

b) Is used by steroid and thyroid hormones (hydrophobic)

c) Receptors contain complexes with three binding sites

d) Binding causes allosteric receptor transformation that binds chromosomal

DNA and activates or inhibits RNA polymerase activity

e) Receptors influence gene expression but require second messengers

4.1) What portion of the pituitary gland (hypophysis) is most anterior?

a) Pars distalis

b) Pars intermedia

c) Pars tuberalis

Page 2: Endocrine Pre-test self test

Endocrine 05May2009

DO NOT DISTRIBUTE - 2 -

d) Pars nervosa

e) Infundibulum

4.2) What portion of the hypophysis contains neurosecretory axons forming the

hypothalamo-hypophyseal tracts?

a) Pars distalis

b) Pars intermedia

c) Pars tuberalis

d) Pars nervosa

e) Infundibulum

4.3) Which of the following describe the embryology of the hypophysis?

a) The anterior lobe grows down from the neuroectoderm forming the Rathke

pouch and the posterior lobe grows up from the oropharynx ectoderm

b) The anterior lobe grows down from the neuroectoderm and the posterior lobe

grows up from the oropharynx ectoderm forming the Rathke pouch

c) The anterior lobe grows up from the oropharynx ectoderm forming the Rathke

pouch and the posterior lobe grows down from the neuroectoderm

d) The anterior lobe grows up from the oropharynx ectoderm and the posterior

lobe grows down from the neuroectoderm forming the Rathke pouch

e) Both the anterior and posterior lobes extend from the floor of the third ventricle

4.4) The superior hypophyseal arteries arising from the internal carotid and posterior

communicating, supply the areas around and including the pituitary stalk. The inferior

hypophyseal arteries, arising from the internal carotid, supply the pars nervosa. The

hypothalamo-hypophyseal portal system serves to carry the neuroendocrine secretions

directly to what portion of the hypophysis, without entering systemic circulation?

a) Pars distalis

b) Pars intermedia

c) Pars tuberalis

d) Pars nervosa

e) Infundibulum

4.5) Nerves entering the anterior lobe of the pituitary gland have vasomotor function and

arise from:

a) Supraoptic nucleus

b) Paraventricular nucleus

c) Hypothalamo-hypophyseal tract

d) Autonomic postganglionic fibers

e) Autonomic preganglionic fibers

Match the pars distalis hormones with the histologic cell description:

4.6) FSH & LH a) Medium sized, oval cells, round central nuclei

4.7) ACTH b) Large polygonal cells with oval nuclei

4.8) TSH c) Medium sized, polygonal cells with round eccentric nuclei

4.9) PRL d) Small oval cells with round eccentric nuclei

4.10) GH e) Large polygonal cells with round eccentric nuclei

4.11) The pars tuberalis, an extension of the anterior lobe along the pituitary stalk, often

shows immunoreactivity for which of the following?

a) ACTH, FSH, LH

b) TSH, PRL, GH

Page 3: Endocrine Pre-test self test

Endocrine 05May2009

DO NOT DISTRIBUTE - 3 -

c) ACTH, GH, LH

d) FSH, PRL, TSH

e) LH, TSH, GH

4.12) Which of the following is a small polypeptide (4,000kDa) and stimulates secretion

of glucocorticoids and gonadocorticoids by the zona fasciculata and zona reticularis?

a) Growth hormone (somatotropin, GH)

b) Prolactin (PRL)

c) Adrenocorticotropin hormone (ACTH)

d) Follicle-stimulating hormone (FSH)

e) Luteinizing hormone (LH)

f) Thyrotropic hormone (TSH)

g) Oxytocin

h) Antidiuretic hormone (ADH, vasopressin)

4.13) Which of the following stimulates spermatogenesis in the testis?

a) Growth hormone (somatotropin, GH)

b) Prolactin (PRL)

c) Adrenocorticotropin hormone (ACTH)

d) Follicle-stimulating hormone (FSH)

e) Luteinizing hormone (LH)

f) Thyrotropic hormone (TSH)

g) Oxytocin

h) Antidiuretic hormone (ADH, vasopressin)

4.14) Which of the following maintains androgen secretion by the Leydig (interstitial)

cells of the testis and regulates final maturation of the ovarian follicle?

a) Growth hormone (somatotropin, GH)

b) Prolactin (PRL)

c) Adrenocorticotropin hormone (ACTH)

d) Follicle-stimulating hormone (FSH)

e) Luteinizing hormone (LH)

f) Thyrotropic hormone (TSH)

g) Oxytocin

h) Antidiuretic hormone (ADH, vasopressin)

4.15) Oxytocin stimulates ejection of milk from the glands and stimulates contraction of

the smooth muscle cells in the pregnant uterus. Which of the following, like oxytocin, is

produced then stored in the neurohypophysis?

a) Antidiuretic hormone (ADH, vasopressin)

b) Adrenocorticotropin hormone (ACTH)

c) Growth hormone (somatotropin, GH)

d) Follicle-stimulating hormone (FSH)

e) Thyrotropic hormone (TSH)

f) Luteinizing hormone (LH)

g) Prolactin (PRL)

5.1) Which of the following hypothalamic regulating hormones inhibits secretion of PRL

by lactotropes?

a) Growth hormone-releasing hormone (GHRH)

b) Dopamine

Page 4: Endocrine Pre-test self test

Endocrine 05May2009

DO NOT DISTRIBUTE - 4 -

c) Corticotropin-releasing hormone (CRH)

d) Somatostatin

e) Thyrotropin-releasing hormone (TRH)

f) Gonadotropin-releasing hormone (GnRH)

5.2) Which of the following hypothalamic regulating hormones inhibits secretion of GH?

a) Growth hormone-releasing hormone (GHRH)

b) Dopamine

c) Corticotropin-releasing hormone (CRH)

d) Somatostatin

e) Thyrotropin-releasing hormone (TRH)

f) Gonadotropin-releasing hormone (GnRH)

5.3) Which of the following stimulates secretion of LH and FSH?

a) Growth hormone-releasing hormone (GHRH)

b) Dopamine

c) Corticotropin-releasing hormone (CRH)

d) Somatostatin

e) Thyrotropin-releasing hormone (TRH)

f) Gonadotropin-releasing hormone (GnRH)

6.1) Which of the following is NOT true of the pineal gland?

a) Located on the posterior wall of the third ventricle

b) Shaped like a flattened pinecone

c) Develops from oropharynx ectoderm

d) Pinealocytes have a large deeply infolded nucleus with one or more nuclei

e) Functions in regulation of daily body rhythm (circadian)

6.2) Which of the following describes the release and action of melatonin?

a) Darkness inhibits the production of melatonin and melatonin stimulates the

neurons in the hypothalamus that produce GnRH

b) Darkness inhibits the production of melatonin and melatonin inhibits the

neurons in the hypothalamus that produce GnRH

c) Daylight inhibits the production of melatonin and melatonin stimulates the

neurons in the hypothalamus that produce GnRH

d) Daylight inhibits the production of melatonin and melatonin inhibits the

neurons in the hypothalamus that produce GnRH

7.1) Which of the following occurs at week 14 of development with regard to the thyroid

gland?

a) Colloid is produced and the gland starts working

b) Thyroid hormone is lost leading to cretinism

c) Follicles develop from endodermal cells

d) Thyroglossal duct descends from endoderm pharynx

e) Thyroglossal duct divides into two lobes at the trachea

7.2) Which of the following is NOT true?

a) Thyroxine (T4) and triiodothyronine (T3) regulate cell basal metabolism and

heat production

b) Calcitonin (thyrocalcitonin) increase blood calcium levels

c) Follicular cells produce T3 and T4

d) Parafollicular cells secrete calcitonin, the physiologic antagonist to PTH

Page 5: Endocrine Pre-test self test

Endocrine 05May2009

DO NOT DISTRIBUTE - 5 -

e) The thyroid follicle produces the colloid thyroglobin, which is the inactive

storage form for the thyroid hormones

7.3) Which of the following is true of tetraiodothyronine (T4)?

a) Formed by the addition of four iodine atoms to thyroglobulin

b) Formed by oxidative coupling of four monoiodotyrosine (MITs)

c) Formed by oxidative coupling of two diiodotyrosines (DITs)

d) Formed by oxidative coupling of two MITs and a DIT

e) Secreted in a T4:T3 ration of 1:20

8.1) Which of the following is true of the parathyroid glands?

a) Superior and inferior glands develop from the third brachial pouch

b) Superior and inferior glands develop from the fourth brachial pouch

c) Superior glands develop from third brachial pouch and inferior from fourth

d) Superior glands develop from fourth brachial pouch and inferior from third

e) All four glands will receive full blood supply without the inferior thyroid artery

8.2) Which of the following is NOT true of parathyroid hormone (PTH)?

a) Promotes calcium release from bone

b) Stimulates calcium reabsorption at distal kidney tubule

c) Increases formation of 1,25-dihydroxycholecalciferol (1,25-OH2)

d) Secreted by chief (principle) cells

e) Absorbed by oxyphil cells

f) Reduces serum phosphate

9.1) Which of the following describes the embryonic development of the adrenal gland?

a) Cortical cells arise from mesoderm; medulla cells originate from endoderm

b) Cortical cells arise from neural crest; medulla cells originate from ectoderm

c) Cortical cells arise from ectoderm; medulla cells originate from neural crest

d) Cortical cells arise from mesoderm; medulla cells originate from neural crest

e) Cortical cells arise from neural crest; medulla cells originate from endoderm

9.2) Which of the following layers of the adrenal gland does NOT receive a direct blood

supply, but receives blood via sinusoids?

a) Capsule

b) Cortex

c) Medulla

Match the adrenal gland secreted hormone with the location:

9.3) Catecholamines (epinephrine) a) Zona glomerulosa

9.4) Mineralcorticoids (aldosterone) b) Zona fasciculata

9.5) Gonadocorticoids (DHEA) c) Zona reticularis

9.6) Glucocorticoids (Cortisol) d) Medullary chromaffin cells

Match the function with the adrenal gland secreted hormone:

9.7) Induces weak masculinizing effect a) Mineralcorticoids

9.8) Aid in controlling electrolyte homeostasis b) Glucocorticoids

9.9) Promote normal metabolism c) Gonadocorticoids

10.1) Which of the following is formed from the ventral bud from the hepatic duct (not

dorsal bud from the foregut)?

a) Uncinate process

b) Body

c) Tail

Page 6: Endocrine Pre-test self test

Endocrine 05May2009

DO NOT DISTRIBUTE - 6 -

d) Anterior head

10.2) What is the function of somatostatin (from D cells)?

a) Activates insulin and glucagon secretions

b) Inhibits insulin and glucagon secretions

c) Activates insulin secretions and inhibits glucagon secretions

d) Activates glucagon secretions and inhibits insulin secretions

10.3) Which of the following is a function of insulin, secreted from beta cells?

a) Stimulates gluconeogenesis

b) Stimulates glycogenolysis

c) Mobilizes fat

d) Phosphorylation and use of glucose

e) Stimulates hepatic lipase

f) Increases blood glucose level

11) A patient presents to the Emergency Room after exposure to radiation from an atomic

bomb. This explosion has likely created radioactive iodine. Flooding which of the

following locations with potassium iodine (e.g. IOSAT pill) would help prevent side

effects?

a) Islet of Langerhans alpha cells

b) Islet of Langerhans beta cells

c) Adrenal cortex

d) Thyroid gland

e) Pineal gland

Endocrine #2 – Physiology: Hypothalamus & Pituitary

1) Which of the following would be involved in a negative feedback signaling loop

located on the same cell?

a) Autocrine signaling

b) Paracrine signaling

c) Endocrine signaling

d) Neurocrine signaling

2) The release of oxytocin to cause contractions during birth is an example of:

a) Autocrine signaling

b) Paracrine signaling

c) Endocrine signaling

d) Neurocrine signaling

3) Pancreatic beta cells secreting insulin that acts on the alpha cells is an example of:

a) Autocrine signaling

b) Paracrine signaling

c) Endocrine signaling

d) Neurocrine signaling

4) The circadian cycle stimulates hormone release (GH and cortisol) in the absence of

external stimulation. This is an example of:

a) Neurotransmitter stimulated hormone release

b) Hormone stimulated hormone release

c) Pulsatile stimulated hormone release

d) Stress stimulated hormone release

Page 7: Endocrine Pre-test self test

Endocrine 05May2009

DO NOT DISTRIBUTE - 7 -

5) Labs are taken from a patient every hour over a 24-hour period. The labs include

cortisol from the blood and GH via binding protein. When would these values most likely

be the highest?

a) Night (2am)

b) Morning (8am)

c) Afternoon (2pm)

d) Evening (8pm)

6) What GLUT transporter is inserted into target cells (muscle, adipose) when insulin is

released from beta cells, which drops blood glucose and causes a negative feedback loop?

a) GLUT1

b) GLUT2

c) GLUT3

d) GLUT4

e) GLUT5

7) The hypothalamus releases a thyroid-releasing hormone (TRH), which stimulates the

anterior pituitary to release a trophic hormone (TSH), which affects the target gland.

Which of the following describes the negative feedback loop for this process?

a) Pituitary hormones reduce the effect of the hypothalamus

b) Pituitary hormones reduce the effect of the hypothalamus and pituitary

c) Target gland hormones reduce the effect of the pituitary

d) Target gland hormones reduce the effect of the pituitary and hypothalamus

e) A & D

f) B & C

g) B & D

8) The hypothalamus releases a cortisol-releasing hormone (CRH), which stimulates the

pituitary to release ACTH, which stimulates the adrenal cortex to release cortisol. Which

of the following describes the negative feedback loop for this process?

a) Pituitary hormones reduce the effect of the hypothalamus

b) Pituitary hormones reduce the effect of the hypothalamus and pituitary

c) Adrenal cortex hormones reduce the effect of the pituitary

d) Adrenal cortex hormones reduce the effect of the pituitary and hypothalamus

e) A & D

f) B & C

g) B & D

9) If the adrenal cortex were disable such that it could not participate in the negative

feedback loop, which of the following would occur?

a) Decreased CRH and increased cortisol

b) Decreased CRH and decreased cortisol

c) Increased CRH and absence of ACTH

d) Decreased ACTH

e) Increased ACTH

10) If the pituitary gland were disabled such that it could not participate in the cortisol

negative feedback loop, which of the following would occur?

a) Increased CRH and increased cortisol

b) Increased CRH and decreased cortisol

c) Decreased CRH and increased cortisol

Page 8: Endocrine Pre-test self test

Endocrine 05May2009

DO NOT DISTRIBUTE - 8 -

d) Decreased CRH and decreased cortisol

11) A 40-year-old truck driver has had difficulty using his side mirrors for traffic behind

him. He has never had any major medical problems in the past. He visits an optometrist,

who determines he has bitemporal hemianopsia, but his vision is 20/20. A head CT scan

reveals slight enlargement of the sella turcica. Which of the following hormones is most

likely being secreted in excessive amounts in this man?

a) Antidiuretic hormone

b) Prolactin

c) ACTH

d) Growth hormone

e) Luteinizing hormone

12) Which of the following targets to the anterior pituitary is inhibitory?

a) CRH (corticotropin-releasing hormone)

b) TRH (thyrotropin-releasing hormone)

c) DA (dopamine acting on prolactin)

d) PRH (prolactin-releasing hormone)

13) Hypothalamic nerve stimulation to the posterior pituitary would lead to all of the

following EXCEPT:

a) Water retention

b) Decreased blood osmolality

c) Uterine contraction

d) Thyroid stimulation

e) Breast milk ejection

14) A patient presents with dilute, high volume urine (polyuria) and extreme thirst

(polydipsia). Which of the following is most likely?

a) No ADH

b) Normal ADH, low sugar

c) Normal ADH, high sugar

d) Excess ADH

15) A 20-pack-year smoker presents with concentrated, high volume urine and expanded

ICR and ECF volume. Which of the following is most likely?

a) No ADH

b) Normal ADH, low sugar

c) Normal ADH, high sugar

d) Excess ADH

16) How does the hypothalamus cause the release of anterior pituitary hormones?

a) Nerve endings directly connect to anterior pituitary

b) Nerve stimulation of the posterior pituitary causes hormone secretion that

activates the anterior pituitary

c) Nerve ending hormones release hormones directly into the anterior pituitary

d) Nerve ending hormones release hormones that follow the systemic blood

supply into the anterior pituitary

e) Nerve ending hormones release hormones that follow the portal blood supply

into the anterior pituitary

17) Which of the following hormones is derived from (created in) the posterior pituitary?

a) Oxytocin

Page 9: Endocrine Pre-test self test

Endocrine 05May2009

DO NOT DISTRIBUTE - 9 -

b) ACTH

c) TSH

d) ADH

e) No hormones

18) Which of the following hormones of the anterior pituitary targets the liver?

a) ACTH (adrenocorticotropic hormone)

b) TSH (thyroid stimulating hormone)

c) GH (growth hormone)

d) FSH (follicle stimulating hormone)

e) PRL (prolactin)

19) Which of the following hormones of the hypothalamic-anterior pituitary axis cause

the release of FSH?

a) TRH

b) CRH

c) GHRH

d) LHRH

e) PRH

20) If the connection between the hypothalamus and anterior pituitary were severed,

which of the following would increase?

a) TSH

b) PRL

c) ACTH

d) LH & FSH

e) GH

21) A middle-aged female patient presents to her physician complaining of weight gain,

weakness, lethargy, and dizziness. She was recently diagnosed with pulmonary

carcinoma and reports that her symptoms began within the last week. A MRI of the lung

region revealed that her lung tumor grew rapidly over the last few weeks. Physical

examination found elevated blood pressure but little to no edema. Laboratory tests

indicate severely elevated antidiuretic hormone (ADH). Further laboratory tests would

indicate:

a) Elevated renal sodium reabsorption

b) Elevated serum sodium concentration

c) Increased atrial natriuretic peptide (ANP) secretion

d) Polyuria

e) Reduced urinary osmolality

22) A 25-year-old male presents to his physician complaining of loss of peripheral vision.

Physical examination was normal, but a CT scan revealed a very large mass on the

anterior pituitary. Laboratory testing could reveal:

a) Elevated serum corticotropin releasing hormone (CRH)

b) Elevated serum growth hormone releasing hormone (GHRH)

c) Elevated serum oxytocin

d) Elevated serum prolactin

e) Elevated urine osmolality

23) A 45-year-old female is diagnosed with an anterior pituitary tumor. Her physician

explains that the best course of treatment would be surgery, but that she would have to

Page 10: Endocrine Pre-test self test

Endocrine 05May2009

DO NOT DISTRIBUTE - 10 -

undergo hormone replacement post-operatively. The hormones that would most likely

have to be replaced include:

a) Antidiuretic hormone (ADH)

b) Corticotropin releasing-hormone (CRH)

c) Growth hormone releasing-hormone (GHRH)

d) Oxytocin

e) Thyroid stimulating hormone (TSH)

Endocrine #3 – Physiology: Thyroid Hormones & Action

1) Which of the following is involved in a positive feedback loop to the hypothalamus?

a) Thyroid releasing factor (TRH)

b) Somatotropin release-inhibiting factor (SRIF)

c) Thyroid stimulating hormone (TSH)

d) Somatostatin

e) T4 & T3

2) Which of the following would occur without a thyroid present?

a) TRH increases and TSH increases

b) TRH increase and TSH decreases

c) TRH decreases and TSH increases

d) TRH decreases and TSH decreases

e) T4, T3, and somatostatin increase

Match the thyroid disorder: T4 TSH TRH

3) Primary hypothyroidism a) Decreased Decreased Increased

4) Pituitary hypothyroidism b) Increased Increased Decreased

5) Hypothalamic hypothyroidism c) Decreased Increased Increased

6) Pituitary hyperthyroidism d) Decreased Decreased Decreased

7) Graves disease (autoimmune) e) Increased Decreased Decreased

8) Calcitonin is produced in which of the following locations?

a) Follicular cells

b) Colloid

c) Thyroid capillary

d) Parafollicular cells

e) None of the above

9) Demineralization and glycosylation of thyroglobulin occurs at what cellular location?

a) Mitochondria

b) Smooth ER

c) Rough ER

d) Golgi apparatus

e) Vesicle before entering colloid

10) Iodine is absorbed against an electrochemical gradient (Na+/I- symporter) and

concentrated above blood levels (iodide trapping) in what location?

a) Follicular cells

b) Colloid

c) Thyroid capillary

d) Parafollicular cells

e) None of the above

Page 11: Endocrine Pre-test self test

Endocrine 05May2009

DO NOT DISTRIBUTE - 11 -

11) Iodide is sent to the colloid via:

a) I-/Cl- exchanger (pedrin)

b) Na+/I- symporter

c) Na+/K+ pump

d) Passive diffusion

e) Active diffusion

12) What is the role of thyroid peroxidase?

a) Releases iodine from thyroglobulin

b) Binds iodine to thyroglobulin

c) Binds two DITs to make T4

d) Binds MIT to DIT to make T3

e) Binds two iodide atoms to make iodine

13) At what location does iodide bind to tyrosine residues on thyroglobulin to form DITs

or MITs, leading to a 2-3 month stored supply of T3 and T4?

a) Follicular cells

b) Colloid

c) Thyroid capillary

d) Parafollicular cells

14) Which of the following is the active form of thyroid hormone?

a) T4

b) T3

c) rT3

15) When T4 diffuses into a cell and is deiodinized to T3, which of the following occurs

prior to gene transcription with RNA polymerase II?

a) T3 binds to a thyroid receptor

b) T3 displaces a corepressor

c) T3 recruits a coactivator

d) A & B

e) All of the above

16) Which of the following actions of thyroid hormone acts synergistically with GH?

a) Increased basal O2 consumption and heat production

b) Increased respiratory rate and minute ventilation

c) Increased cardiac output (HR & SV)

d) Increased linear bone growth

e) Increased glucose absorption

17) Which of the following, along with norepinephrine, decreases with thyroid hormone

action?

a) Hypothalamus SNS center stimulation

b) Red blood cell mass and O2 capacity

c) Total peripheral resistance

d) Cholesterol turnover (LDL decrease)

e) Epinephrine activity (lipolysis)

f) Sensory awareness and memory

18) Which of the following would occur in a patient who developed antibodies that are

bound to TSH-R and competitively inhibited TSH (Hashimoto disease)?

a) TSH increases

Page 12: Endocrine Pre-test self test

Endocrine 05May2009

DO NOT DISTRIBUTE - 12 -

b) TRH decreases

c) T4 production increases

d) T3 production increases

19) A patient develops antibodies that bind TSH-R and cause hyperstimulation (Graves

disease). Which of the following is most likely?

a) Increased T4, increased TSH, increased TRH

b) Increased T4, increased TSH, decreased TRH

c) Increased T4, decreased TSH, decreased TRH

d) Increased T4, decreased TSH, increased TRH

e) Decreased T4, increased TSH, decreased TRH

20) A 32-year-old woman complains of excessive tiredness, weight gain, constipation

and breast discharge. Laboratory results indicate that serum thyroid stimulating hormone

(TSH) was markedly decreased. Additional physical examination and laboratory testing

would most likely reveal:

a) Decreased thyrotropin-releasing hormone (TRH)

b) Increased respiration rate

c) Increased TRH

d) Increased triiodothyronine (T3)

e) Increased T4

Three patients with hypothyroidism arrive at the laboratory for a thyroid-releasing

hormone (TRH) test. During this test, TRH was injected at time 0 and plasma TSH was

measured for 3 hours. The following graph was created for the findings:

21) What disorder does Patient A have?

a) Thyroid

b) Hypothalamus

c) Pituitary

22) What disorder does Patient B have?

a) Thyroid

b) Hypothalamus

c) Pituitary

23) What disorder does Patient C have?

a) Thyroid

b) Hypothalamus

c) Pituitary

Page 13: Endocrine Pre-test self test

Endocrine 05May2009

DO NOT DISTRIBUTE - 13 -

24) An adult female patient presented to her physician complaining that she feels weak

and fatigue all the time. She reports that she and her husband are getting ready to be

divorced, and that she is feeling stressed. Laboratory tests for thyroid function revealed

no abnormalities. The physician, however, prescribed her T3. If further testing was done

2 days after beginning treatment, the findings would most likely include:

a) Elevated thyroid stimulating hormone (TSH)

b) Increased iodide uptake

c) Increased thyroglobulin synthesis

d) No change in iodide uptake

e) Reduced T4

25) An experimental animal was created to determine the effects of thyroid stimulating

hormone (TSH) deficiency on growth and development. Scientists genetically altered a

mouse model by mutating the gene responsible for TSH receptors. A colony was created

and called the TSH receptor knockout (TSH-R-KO) mouse strain. The predicted

phenotype of TSH-R-KO offspring would include:

a) Above average intellectual capabilities

b) Dwarfism

c) Elevated basal metabolic rate

d) Elevated body temperature

e) Elevated growth hormone (GH) secretion

26) A female patient presents to her physician complaining of agitation, weight loss,

increased appetite, abnormal menstrual cycles, and sleep disturbances. Physical

examination reveals that her skin is moist and she has a temperature of 100°C. She

reports that her skin itches sometimes and examination reveals pretibial myxedema. A

triiodothyronine (T3) resin uptake test (T3RU) revealed increased T3 resin uptake. The

physician diagnoses her with Graves’ disease. Further laboratory tests would reveal:

a) Decreased T3 levels

b) Decreased T4 levels

c) Elevated thyroid binding globulin concentration

d) Elevated thyroid stimulating hormone

e) Saturation of thyroid binding globulin (TBG) binding sites

Endocrine #4 – Physiology: Parathyroid Regulation & Bone Remodeling

1) Where is calcium mainly stored in the body?

a) Bound to albumin

b) Bone and teeth

c) Kidneys

d) Ionized in serum

e) Non-ionized in serum (complexed)

f) Muscle

2) What calcium turnover process results in the loss of the most calcium?

a) Bone remodeling

b) Kidney filtering

c) ECF secretion

d) Intestinal excretion

e) Exchangeable pool

Page 14: Endocrine Pre-test self test

Endocrine 05May2009

DO NOT DISTRIBUTE - 14 -

3) Where is phosphate mainly stored in the body?

a) Teeth

b) Muscle

c) Bone

d) Kidneys

e) Blood

4) What phosphate turnover process results in the loss of the most phosphate?

a) Bone remodeling

b) Kidney filtration

c) Exchangeable pool

d) Intestinal secretion

e) Soft tissue remodeling

5) Which of the following describes the action of parathyroid hormone (PTH)?

a) Increased Ca++ and bone reabsorption, increased phosphate reabsorption

b) Increased Ca++ and bone reabsorption, decreased phosphate reabsorption

c) Decreased Ca++ and bone reabsorption, increased phosphate reabsorption

d) Decreased Ca++ and bone reabsorption, decreased phosphate reabsorption

6) Which of the following describes the action of vitamin D (intestine & bone)?

a) Increased Ca++ and bone absorption, increased phosphate absorption

b) Increased Ca++ and bone absorption, decreased phosphate absorption

c) Decreased Ca++ and bone absorption, increased phosphate absorption

d) Decreased Ca++ and bone absorption, decreased phosphate absorption

7) Which of the following calcium changes would occur in a patient who has their

parafollicular thyroid cells removed?

a) Increased calcium stores

b) Decreased calcium stores

c) Increased serum calcium

d) Decreased serum calcium

e) No change in calcium

8) A patient is found to have a PTH tumor. Which of the following sequela may occur?

a) Difficulty urinating

b) Goiter

c) Soft tissue pain

d) Bone fracture

e) Carotid bruit

9) What is the action of calcitonin?

a) Activate osteoblasts

b) Deactivate osteoblasts

c) Activate osteoclasts

d) Deactivate osteoclasts

10) Which of the following is NOT true of parathyroid hormone?

a) Produced by chief cells of the parathyroid gland

b) Decreases ionized plasma Ca++ levels

c) Increases plasma phosphate levels

d) Inhibited by 1,25-OH2-D via negative feedback

e) Decreased vitamin D production and bone resorption

Page 15: Endocrine Pre-test self test

Endocrine 05May2009

DO NOT DISTRIBUTE - 15 -

11) A patient presents with complains of lethargy. Testing reveals a likely PTH-releasing

tumor. If phosphate is the likely cause of the lethargy, how is PTH affecting the kidneys?

a) Increased reabsorption of phosphate at the proximal tubule

b) Decreased reabsorption of phosphate at the proximal tubule

c) Increased reabsorption of phosphate at the distal tubule

d) Decreased reabsorption of phosphate at the distal tubule

12) What affect does PTH have on calcium in the kidneys?

a) Increased reabsorption of calcium at the proximal tubule

b) Decreased reabsorption of calcium at the proximal tubule

c) Increased reabsorption of calcium at the distal tubule

d) Decreased reabsorption of calcium at the distal tubule

13) Which of the following would be seen in a patient with hypoparathyroidism?

a) Hypercalcemia

b) Hypercalciuria

c) Renal stones

d) Hyperphosphatemia

e) Decreased bone mass

14) Vitamin D (D2 in plants/yeast, D3 in diet/skin) must be hydroxylated to its active

form, 1,25-OH2-D. Where does the first hydroxylation occur?

a) Liver

b) Blood

c) Kidney proximal tubule

d) Kidney distal tubule

e) Bone and teeth

15) Where does 1alpha-hydroxylase act?

a) Liver

b) Blood

c) Kidney proximal tubule

d) Kidney distal tubule

e) Bone and teeth

16) Which of the following describes the activity of 1,25-OH2D3?

a) Increases calcium absorption in the intestine

b) Increases calcium excretion from the intestine

c) Increases proximal tubule calcium reabsorption

d) Decreases basolateral membrane Ca++ pumps

e) Decreases calbindin synthesis

17) How is calcium transported from the basolateral side to the interstitial space?

a) Ca++/H+ ATPase

b) 3Na+/Ca++ exchanger

c) Calbindin

d) A & B

e) A & C

18) A child presents with leg-bone deformity. Blood testing shows high levels of vitamin

D in the blood. Which of the following is most likely?

a) Type I rickets

b) Type II rickets

Page 16: Endocrine Pre-test self test

Endocrine 05May2009

DO NOT DISTRIBUTE - 16 -

c) Renal stones

d) Excess 1,25-OH2D3

e) Calcitonin deficiency

19) A man presents to his physician complaining of muscle cramping, irritability and

tingling in the fingers and toes. Laboratory tests indicate that he is hypocalcemic and

hyperphosphatemic. All other measures are unremarkable. Further laboratory tests would

indicate:

a) Elevated calcitonin levels

b) Hyperparathyroidism

c) Hypoparathyroidism

d) Vitamin D deficiency

e) Vitamin D excess

20) A female patient presents complaining of pain in the lower right side of her back,

nausea, constipation, fatigue, muscle weakness, and lethargy. An MRI of the

thyroid/parathyroid region revealed a small tumor on the parathyroid gland causing

parathyroid over-activation. Further laboratory tests would indicate:

a) High serum phosphate, high serum calcium, and high serum parathyroid

hormone (PTH)

b) High serum phosphate, low serum calcium, and low serum PTH

c) Low serum phosphate, high serum calcium, and high serum PTH

d) Low serum phosphate, high serum calcium, and low serum PTH

e) Low serum phosphate, low serum calcium, and high serum PTH

21) A female patient with renal disease undergoes laboratory tests to measure plasma

ions and renal function. Based on the results, her physician diagnoses her with a rare

autoimmune disorder that is causing progressive kidney destruction. This destruction will

probably also result in the impairment in the conversion of:

a) 1,25-dihydroxycholecalciferol to 25-hydroxycholecalciferol

b) 25-hydroxycholecalciferol to 1,25-dihydroxycholecalciferol

c) Cholesterol to 7-dehydrocholesterol

d) Vitamin D3 to 25-hydroxycholecalciferol

e) Vitamin D3 to Vitamin D2

22) Which of the following is true?

a) Most of bone mass is cortical bone and remodeling mostly affects cortical bone

b) Most of bone mass is cortical bone and remodeling mostly affects spongy bone

c) Most of bone mass is spongy bone and remodeling mostly affects cortical bone

d) Most of bone mass is spongy bone and remodeling mostly affects spongy bone

23) Which of the following is contained within the osteoid?

a) Ground substance

b) Type I collagen

c) Proteoglycans

d) Minerals (CaPO4, Na, Carbonate, Mg)

e) All of the above

24) Which of the following is stimulated by cytokines and inhibited by calcitonin?

a) Osteoblasts

b) Osteocytes

c) Osteoclasts

Page 17: Endocrine Pre-test self test

Endocrine 05May2009

DO NOT DISTRIBUTE - 17 -

d) Osteocalcin

e) Osteonectin

25) Annexins are secreted by which of the following in order to recruit more of their

own?

a) Osteoblasts

b) Osteocytes

c) Osteoclasts

26) What paracrine signaling is accomplished during bone remodeling?

a) Osteocytes signal osteoblasts

b) Osteocytes signal osteoclasts

c) Osteoblasts signal osteoclasts

d) Osteoblasts signal osteocytes

e) Osteoclasts signal osteoblasts

27) PTH stimulates bone resorption by activating:

a) Osteoblasts

b) Osteocytes

c) Osteoclasts

28) Which of the following people would have the most bone mass?

a) Teenage soccer player

b) Young adult weight lifter

c) Female undergoing menopause

d) Adult male jogger

e) Elderly bedridden patient

Endocrine #5 – Physiology: Adrenal Hormones

Match the hormone with the primary location of production:

1) Catecholamines a) Adrenal cortex, zona glomerulosa

2) Aldosterone b) Adrenal cortex, zona fasciculata

3) Cortisol c) Adrenal cortex, zona reticularis

4) Androgens d) Adrenal medulla

5) A patient with hyperaldosteronism would likely have:

a) Low ANP levels

b) Low 18-OH-DOC levels

c) Hypercalcemia

d) Hypomagnesemia

e) Hypokalemia

6) Which of the following would lead to an overall decrease in sodium and water

excretion due to activation of ADH in the renin-angiotensin-aldosterone system (RAAS)?

a) Increased blood pressure

b) Decreased blood hematocrit

c) Increased blood osmolarity

d) Decreased blood osmolality

e) Increased sodium retention

7) Angiotensin II is converted to aldosterone at what location?

a) Brain

b) Adrenals

Page 18: Endocrine Pre-test self test

Endocrine 05May2009

DO NOT DISTRIBUTE - 18 -

c) Lungs

d) Liver

e) Kidneys

8) What is the main function of aldosterone?

a) Decrease sodium reabsorption

b) Increase K+ secretion

c) Maintain ECF volume

d) Increase H+ secretion

e) Deplete ICF volume

9) Cortisol releasing factor (CRF) from the hypothalamus would lead to all of the

following EXCEPT:

a) Increased growth

b) Increased blood pressure

c) Increased arousal

d) Increased SNS

e) Decreased reproductive hormones

10) Which of the following would inhibit ACTH?

a) Stress

b) Depression

c) Anxiety

d) Cortisol

e) Alpha-agonists

11) Which of the following would lead to a higher setpoint for cortisol due to an over-

ride of the negative feedback loop?

a) Sleeping during the day

b) Strenuous exercise

c) Constant stress

d) High fat diet

e) Decreased CRH

12) Glucocorticoids (e.g. cortisol) inhibit which of the following via a negative feedback

loop?

a) Hypothalamus

b) Corticotroph (anterior pituitary)

c) Adrenal cortex

d) A & B

e) A & C

Match the glucocorticoid disorder: Plasma cortisol Plasma ACTH

13) Primary hypercortisolism a) Increased Increased

14) Secondary hypercortisolism b) Decreased Increased

15) Primary hypocortisolism c) Increase Decreased

16) Secondary hypocortisolism d) Decreased Decreased

17) Which of the following is NOT an effect of glucocorticoids?

a) Insulin resistance

b) Lipogenesis in the trunk

c) NPY inhibition

d) Lipolysis in the extremities

Page 19: Endocrine Pre-test self test

Endocrine 05May2009

DO NOT DISTRIBUTE - 19 -

e) Muscle protein turnover and gluconeogenesis

18) Which of the following is NOT an effect of glucocorticoids?

a) Decrease in muscle mass and strength

b) Decreased bone formation and increased resorption

c) Thinning of skin and capillary walls

d) Decreases glomerular filtration rate

e) Decreased memory

19) Which of the following is NOT an effect of glucocorticoids?

a) Inhibits arachidonic acid synthesis

b) Decreases neutrophil number but increases effectiveness

c) Decreases fibroblast proliferation

d) Stimulates T-cell apoptosis

e) Inhibits IL, INF-gamma, COX-2 expression

20) Which of the following is true of androgens?

a) Most of male androgens exist in the adrenals

b) Female ovaries provide most of their androgens

c) A castrated male can use the adrenals to compensate for testosterone loss

d) Adrenal androgen is converted to testosterone in the female pubic and axillary

areas

e) Adrenal androgens are of great importance to a physiologic male

21) Scientists were conducting an experiment to look at the effect of surgical ablation on

kidney function. After doing 40 ablation surgeries, their student was tired and anxious to

finish. On his 41st surgery he accidentally removed a large section of the outer layer of

the adrenal gland. If the animal were allowed to recover from surgery, hormonal

measurements would probably indicate:

a) Decreased adrenocorticotropic hormone (ACTH) secretion

b) Decreased aldosterone production

c) Decreased androgen production

d) Decreased catecholamine production

e) Decreased stress-induced cortisol secretion

22) In an experiment designed to characterize the hypothalamic-pituitary-adrenal axis in

rodents, plasma adrenocorticotropic hormone (ACTH) and corticosterone were measured

in several rats following various surgical and pharmaco-

logical manipulations. The following graph plots the

relative hormone levels for each animal. Hormonal profiles

for each group reveal:

a) Group A may have adrenal hyperplasia

b) Group A may have primary adrenal insufficiency

c) Group B may have hypothalamic deficiency

d) Group B may have pituitary deficiency

e) Group C may have adrenal hyperplasia

23) A male patient presents to his physician complaining of muscle weakness and

abnormal heartbeat he describes as “beating out of tune.” He was previously been

diagnosed with Type 1 (insulin-dependent) diabetes mellitus, but reports strict adherence

to insulin treatments and eats a low-salt and low-carbohydrate diet. Laboratory tests

Page 20: Endocrine Pre-test self test

Endocrine 05May2009

DO NOT DISTRIBUTE - 20 -

measuring serum electrolytes revealed low sodium and elevated potassium levels. Further

laboratory tests would reveal:

a) Elevated ACTH secretion

b) Elevated aldosterone secretion

c) Elevated glucocorticoid secretion

d) Reduced ACTH secretion

e) Reduced renin secretion

24) A 49-year-old male presents to his physician complaining of a persistent “dry” cough,

muscle weakness, and reduced libido. Physical examination reveals abnormal weight gain

in the back, abdomen, and face and abdominal stria. He reports that he has smoked 2

packs of cigarettes per day for the last 30 years. Laboratory tests revealed elevated serum

glucose, a positive dexamethasone suppression test, and a hormone-secreting bronchial

carcinoma. Further laboratory tests would most likely reveal that the hormone secreted by

the tumor is:

a) Adrenocorticotropic hormone (ACTH)

b) Cortisol

c) Glucagon

d) Growth hormone

e) Thyroxine

25) A male patient is experiencing adrenal insufficiency and cannot produce adequate

amounts of cortisol, either basal or stress-induced. Under basal conditions metabolism is

not greatly affected, but he experiences severe weakness when confronted with a stressor.

Laboratory tests conducted during periods of stress would likely indicate:

a) Decreased glucose uptake in muscles and adipose

b) Hypoglycemia

c) Increased circulating amino acids

d) Increased gluconeogenesis

e) Increased lipoysis

26) Where is norepinephrine mostly produced?

a) Preganglionic sympathetic fibers

b) Postganglionic sympathetic fibers

c) Preganglionic parasympathetic fibers

d) Postganglionic parasympathetic fibers

27) Which of the following actions is decreased by epinephrine and norepinephrine?

a) Insulin action

b) Glucagon secretion

c) BMR and thermogenesis

d) Gluconeogenesis

e) Glycogenolysis

28) Which of the following actions is decreased by epinephrine and norepinephrine?

a) Heart rate

b) Renin secretion

c) Skin blood flow

d) Systolic pressure

e) Cardiac contractility

f) Bronchodilation

Page 21: Endocrine Pre-test self test

Endocrine 05May2009

DO NOT DISTRIBUTE - 21 -

29) A male patient presents to his physician complaining of feelings of “nervousness”

followed by fatigue. Subsequent tests revealed an adrenal tumor in the medullary region.

Further laboratory testing would most likely reveal:

a) Decreased circulating free fatty acids

b) Decreased circulating glucose levels

c) Decreased circulating insulin levels

d) Increased glycogenesis

e) Increased lipogenesis

Endocrine #6 – Physiology: Hormones Involved in Food Intake & Energy Storage

Match the hypothalamic location with the function:

1) Satiety a) Lateral nuclei

2) Regulation of feeding b) Ventromedial nuclei

3) Hunger c) Paraventricular nucleus (PVN)

4) Multi-hormone action d) Dorsomedial nuclei (DMN)

5) Increase eating e) Arcuate nuclei (ACN)

6) Which of the following components of the arcuate nucleus decreases food intake and

increases energy expenditure?

a) Alpha-MSH (melanocyte-stimulating hormone)

b) NPY (neuropeptide Y)

c) CART (cocaine- and amphetamine-related transcripts)

d) AGRP (agouti-related protein)

e) A & C

f) B & D

7) POMC neurons release alpha-MSH, which acts on MCR-3 and MCR-4 melanocortin

receptors in the paraventricular nucleus. Which of the following would occur if a patient

had an MCR-4 mutation or who receives MCR-4 antagonist (AGRP)?

a) Addiction to cocaine

b) Rapid weight loss

c) Early satiety

d) Obesity

e) Intolerance to amphetamines

8) Which of the following decreases POMC activation and is the most associated with

increased appetite (hunger)?

a) NPY

b) Alpha-MSH

c) CART

d) AGRP

9) Antagonizing which of the following could be a helpful treatment for an obese patient

whose weight problem is caused by Leptin?

a) MCH (melanin-concentrating hormone)

b) CART (cocaine- and amphetamine-related transcripts)

c) Orexin (hypocretin)

d) NPY (neuropeptide Y)

e) CB1 (cannabinoid)

10) Which of the following would increase appetite and decrease ACTH levels?

Page 22: Endocrine Pre-test self test

Endocrine 05May2009

DO NOT DISTRIBUTE - 22 -

a) MCH (melanin-concentrating hormone)

b) CART (cocaine- and amphetamine-related transcripts)

c) Orexin (hypocretin)

d) NPY (neuropeptide Y)

e) CB1 (cannabinoid)

11) A deficiency of orexin (hypocretin) would lead to:

a) Malnutrition

b) Neoplasm

c) Narcolepsy

d) Diabetes

e) Agitation

12) Which of the following short-term regulators that decrease eating responds to fat

entering the duodenum?

a) Peptide YY

b) Glucagon-like peptide

c) Cholecystokinin

d) Insulin

13) A patient with Prader-Willi is found to have high levels of Ghrelin. What affect does

this have?

a) Hunger

b) Double vision

c) Easy bruising

d) Hyperthyroidism

e) Hypogonadism

14) Which of the following would NOT occur in the presence of Leptin?

a) Decreased NPY and AGRP

b) POMC inhibition

c) CRH increase

d) SNS increase

e) Insulin decrease

15) The Zucker Diabetic Fatty (ZDF) rat has which of the following disorders?

a) Childhood over-nutrition to increase fat cells

b) Hypothalamic lesion

c) Hypophyseal tumor

d) MCR gene mutation

e) Leptin receptor mutation

16) Which of the following is the recommended primary obesity treatment?

a) Physical exercise

b) Amphetamines

c) Altering lipid metabolism

d) Gastric bypass

e) Gastric banding

17) A mutation in POMC would cause which of the following?

a) Increased food intake and weight gain

b) Increased food intake but weight loss

c) Decreased food intake and weight loss

Page 23: Endocrine Pre-test self test

Endocrine 05May2009

DO NOT DISTRIBUTE - 23 -

d) Decreased food intake but weight gain

18) What is the mechanism by which an AGRP gene mutation decreases food intake?

a) Antagonizes MCR-3 and MCR-4 receptors

b) Cause release of alpha-MSH

c) SNS activation to the nucleus tractus solitarius

d) Decreases POMC activation

e) Leptin antagonism

19) What is the mechanism by which an NPY gene mutation decreases food intake?

a) Antagonizes MCR-3 and MCR-4 receptors

b) Cause release of alpha-MSH

c) SNS activation to the nucleus tractus solitarius

d) Decreases POMC activation

e) Leptin antagonism

Endocrine #7 – Physiology: Growth

1) Which of the following would lead to an increase in growth hormone (GH)?

a) Caloric restriction

b) Increased amino acids

c) Increased glucose or free fatty acids

d) Exercise or stress

e) Ghrelin

2) Which of the following inhibits GH by acting on GHRH?

a) Somatostatin

b) Cortisol

c) Old-age

d) Obesity

e) Pregnancy

3) A woman presents with acromegaly and breast discharge. Which of the following

hormones would likely be high?

a) Prolactin

b) Oxytocin

c) ACTH

d) GH

e) ADH

4) The JAK-STAT pathway involves taking dimerized GH and transcribing it to IGF-1

(insulin-like growth factor). Where does this mainly occur?

a) Liver

b) Long bone

c) Pituitary

d) Kidney

e) Hypothalamus

5) Which of the following is a function of IGF-1, not GH?

a) Gluconeogenesis

b) Bone and tissue growth

c) Insulin resistance

d) Protein synthesis

Page 24: Endocrine Pre-test self test

Endocrine 05May2009

DO NOT DISTRIBUTE - 24 -

e) Lipolysis

6) Which of the following promotes the closure of long bone epiphyseal plates?

a) GH

b) IGF-1

c) Estrogen & testosterone

d) Insulin & thyroid hormones

7) Which of the following would NOT occur in an adult with high levels of growth

hormone?

a) Prominent jaw

b) Prominent brow

c) Large hands and feet

d) Increase height

e) Large nose

8) Hypothyroidism in infancy would lead to what growth effects?

a) Developmental retardation

b) Large long bones

c) Leg length discrepancies

d) Prominent jaw and nose

e) Organomegaly, especially heart

9) Which of the following would occur in a patient with congenital growth hormone

receptor dysfunction?

a) Dwarfism with low GH levels and secondary sex characteristics present

b) Dwarfism with high GH levels and secondary sex characteristics present

c) Dwarfism with low GH levels and no secondary sex characteristics

d) Dwarfism with high GH levels and no secondary sex characteristics

10) Which of the following would be seen in a patient with excess growth hormone?

a) Hypoglycemia and hypergonadism

b) Hyperglycemia and hypergonadism

c) Hypoglycemia and hypogonadism

d) Hyperglycemia and hypogonadism

11) A 42-year-old male visits a local physician for a physical exam. He reports that this is

his first physical in 5 years, and that he is concerned about changes in his appearance

over the last couple years. He explains that his feet and hands have grown larger, his jaw

is protruding, and his face is “looking strange.” He reports slight difficulty breathing,

frequent headaches, fatigue and weakness, impotence, and excessive sweating.

Laboratory tests indicate elevated insulin-like growth factor-1 (IGF-1), and an MRI scan

revealed a 1.5 cm mass on the anterior pituitary. Further physical examination and

laboratory tests may indicate:

a) Decreased hypothalamic somatostatin secretion

b) Decreased protein synthesis

c) Hypoglycemia

d) Insulin resistance

e) Muscle atrophy

12) A 25-year-old male presents to his physician for an annual physical. He was

previously diagnosed with a growth hormone (GH)-secreting tumor on his anterior

Page 25: Endocrine Pre-test self test

Endocrine 05May2009

DO NOT DISTRIBUTE - 25 -

pituitary, for which he is taking a somatostatin analog. Laboratory tests, however, reveal

elevated GH levels. Further laboratory tests would most likely reveal:

a) Decreased gluconeogenesis

b) Increased lipolysis

c) Increased long bone growth

d) Increased insulin action

e) Decreased insulin levels

13) A young adult female patient visits her physician for a physical exam. The physician

observes distinct changes in her appearance that suggest excess growth hormone (GH)

action. Her feet and hands appear larger than expected, and her jaw protrudes more than

normal. Laboratory tests indicate elevated insulin-like growth factor-1 (IGF-1), and an

MRI scan revealed a 1.5 cm mass on the anterior pituitary. Further physical examination

and laboratory tests may indicate:

a) Decreased circulating free fatty acids

b) Decreased hypothalamic somatostatin secretion

c) Decreased lean muscle mass

d) Increased lipogenesis

e) Hyperglycemia

14) A physician notices abnormal growth patterns in a young male child during an annual

physical. The child was severely below normal on standardized growth tests for both

height and weight. All other aspects of the physical examination were unremarkable.

Laboratory tests indicate normal thyroxine and triiodothyronine and decreased levels of

insulin-like growth factor-1. Further laboratory tests would indicate:

a) A decrease in insulin-like growth factor binding proteins (IGFBP)

b) A decrease in thyroid stimulating hormone (TSH)

c) An increase in growth hormone (GH) secretion

d) An increase in IGFBP

e) An increase in thyroid stimulating hormone (TSH)

15) An elderly male presents to his physician to discuss with his physician a new drug

that he saw on the television. He reported that he was always in very good shape but is

now “soft and flabby.” He explained that he saw a commercial about human growth

hormone (GH) administration that was described as a “fountain of youth” and he wanted

to see if he could begin taking it. Laboratory tests revealed that the patient had reduced

GH levels, and the doctor agreed to prescribe the GH replacement for him. The most

likely effect of GH replacement would be:

a) Decreased collagen synthesis

b) Decreased lipolysis

c) Decreased protein synthesis

d) Increased glucose uptake in muscle and adipose

e) Increased plasma glucose

Endocrine #8 – Physiology: Endocrine Pancreas

1) Which of the following cells secrete amylin and pancreastatin?

a) Alpha cells

b) Beta cells

c) Delta cells

Page 26: Endocrine Pre-test self test

Endocrine 05May2009

DO NOT DISTRIBUTE - 26 -

d) PP cells

2) C peptide has a long serum half-life so it can be helpful in determining past levels of

which of the following?

a) Insulin

b) Glucagon

c) Somatostatin

d) Pancreatic polypeptide

e) Glucose

3) Which of the following cells is the first to receive arterial blood supply?

a) Alpha cells

b) Beta cells

c) Delta cells

4) Which of the following inhibits insulin?

a) Eating

b) Glucagon

c) Intestinal hormones

d) Glucose

e) Leptin

5) Which of the following describes the insulin response associated with glucose

injection?

a) Monophasic with an initial rapid increase due to intestinal hormones

b) Monophasic with a gradual increase due to intestinal hormones

c) Biphasic with an initial rapid increase due to intestinal hormones and a

secondary increase due to beta cells

d) Biphasic with an initial rapid increase due to beta cells and a secondary

increase due to intestinal hormones

e) Monophasic with a gradual increase due to beta cells

6) Insulin causes the insertion of what glucose transporter, leading to glucose crossing

membranes to be converted to glycogen in adipose tissue and skeletal muscle?

a) GLUT1

b) GLUT2

c) GLUT3

d) GLUT4

e) GLUT5

7) Insulin causes a decrease in which of the following within the liver?

a) Glycogenolysis

b) Glycolysis

c) Glycogenesis

d) Protein synthesis

e) Lipogenesis

8) Which of the following stimulates glucagon secretion?

a) Somatostatin

b) Insulin

c) Hypoglycemia

d) Secretin

9) What is the major action of glucagon in the pancreas?

Page 27: Endocrine Pre-test self test

Endocrine 05May2009

DO NOT DISTRIBUTE - 27 -

a) Increased glycogenolysis

b) Gluconeogenesis

c) Increased insulin

d) Lipolysis

e) Ketogenesis

10) Which of the following would cause the insulin to glucagon ratio to increase the most

(>10.0 from normal of 2.0?

a) Pure protein meal

b) Carbohydrate loading

c) Pure fat meal

d) Prolonged exercise

e) Fasting

11) Glucagon-like peptide-1 (GLP-1) is released from the intestines and has what action?

a) Increase insulin and increase glucagon

b) Increase insulin and decrease glucagon

c) Decrease insulin and increase glucagon

d) Decrease insulin and decrease glucagon

12) Which of the following is an action of somatostatin?

a) Inhibits insulin

b) Inhibits glucagon

c) Inhibits GI motility and secretions

d) Inhibits glucose and triglyceride absorption

e) All of the above

13) Which of the following is regulated by hypoglycemia, inhibited by hyperglycemia,

and is used diagnostically as a marker for an islet cell tumor?

a) Insulin

b) Glucagon

c) Somatostatin

d) Pancreatic polypeptide

e) Glucose

14) Which of the following is NOT a symptom of Type-1 (insulin dependent) or Type-2

(non-insulin dependent) diabetes mellitus?

a) Weight gain

b) Polydipsia

c) Polyuria

d) Increased food consumption

15) Which of the following is associated with Type-1 diabetes more so than Type-2?

a) Genetic component

b) Obesity as a cause

c) Native American race

d) Autoimmune reaction

e) Sulfonylurea drugs for treatment

16) A middle aged male presented to his physician for an annual physical. Routine testing

revealed elevated plasma glucose levels. Further laboratory testing revealed elevated

insulin levels and severely elevated glucagon levels. Following an MRI, he was

Page 28: Endocrine Pre-test self test

Endocrine 05May2009

DO NOT DISTRIBUTE - 28 -

diagnosed with a pancreatic !-cell tumor. The elevated glucose levels in this patient is

most likely due to:

a) Decreased glycogenolysis

b) Decreased lipogenesis

c) Increased gluconeogenesis

d) Increased lipolysis

e) Increased hepatic glycogenesis

17) A 37-year-old pregnant woman visits her physician for a prenatal visit. She was in

her 13th week of gestation and had been healthy with a normal pregnancy thus far.

Urinalysis revealed elevated glucose levels and blood tests revealed elevated insulin. She

has no history of diabetes, although her mother developed Type 2 (non-insulin

dependent) diabetes later in life. Her elevated glucose levels are most likely due to:

a) Autoimmune pancreatic alpha cell destruction

b) Autoimmune pancreatic beta cell destruction

c) Elevated growth hormone (GH) secretion

d) Increased carbohydrate ingestion

e) Insulin resistance

18) An experiment was designed to determine the effects of a high-fat diet on

carbohydrate metabolism. Rats in group A were

fed a special formulated diet high in fat, and rats in

group B were fed an isocaloric low-fat diet. These

diets were fed to the rats for 2 weeks and a fasting

glucose tolerance test was performed. The

following graph illustrates the results for group A

(Time 0 is glucose administration). The condition

that would most likely produce these results is:

a) Excess glucagon release

b) Increased glycogenesis

c) Increased glycogenolysis

d) Type-1 (insulin dependent) diabetes mellitus

e) Type-2 (noninsulin dependent) diabetes mellitus

Endocrine #9 – Pharmacology: Hypothalamic & Pituitary Hormones

1) Which of the following is located directly superior to the sella turcica?

a) Hypothalamus

b) Pituitary

c) Pons

d) Cingulate gyrus

e) Medulla

2.1) Which of the following hormones released from the anterior pituitary does NOT

stimulate the production of hormones by a peripheral endocrine gland or the liver?

a) FSH

b) LH

c) ACTH

d) TSH

e) PRL

Page 29: Endocrine Pre-test self test

Endocrine 05May2009

DO NOT DISTRIBUTE - 29 -

2.2) Posterior pituitary hormones (Oxytocin, ADH) are released from neurons that arise

from what area?

a) Preoptic nucleus

b) Supraoptic nucleus

c) Paraventricular nucleus

d) Cingulate gyrus

e) Solitary nucleus

3.1) Dopamine from the hypothalamus inhibits which of the following?

a) Growth hormone (GH, somatotropin)

b) Thyroid-stimulating hormone (TSH)

c) Adrenocorticotropin (ACTH)

d) Follicle-stimulating hormone (FSH)

e) Luteinizing hormone (LH)

f) Prolactin (PRL)

3.2) Which hypothalamic hormone is used rarely to differentiate Cushing disease from

ectopic secretion?

a) Growth hormone-releasing hormone (GHRH)

b) Thyrotropin-releasing hormone (TRH, protirelin)

c) Corticotropin-releasing hormone (CRH)

d) Gonadotropin-releasing hormone (GnRH)

e) Dopamine

4) Which of the following conditions for which human growth hormone (hGH) treatment

is indicated is considered the most controversial?

a) Prader-Willi syndrome

b) Turner syndrome

c) Idiopathic short stature

d) None of the above

5) A patient with infertility is undergoing fertility treatment in order to conceive. On day

three of her menstrual cycle, she is started on daily injections of an FSH preparation,

which continue for about 7-12 days. Since exogenous gonadotropins are being used to

stimulate follicular development, they are administered in conjunction with either a

continuous GnRH agonist (down-regulates receptors) or a GnRH antagonist. Why?

a) To prevent a luteal surge

b) To reduce endometrial thickness

c) To increase FSH levels

d) To reduce serum estradiol levels

e) To cause rapid ovulation

6) Which of the following are D2 agonists and thus are useful for hyperprolactinemia?

a) Fenoldopam

b) Bromocriptine and cabergoline

c) Metoclopramide, droperidol, and domperidone

d) Clozapine, risperidone, and olanzapine

e) Amoxapine

7) A post-partum woman is having continual uterine bleeding. Which of the following

could be given to help reduce hemorrhage?

a) Oxytocin

Page 30: Endocrine Pre-test self test

Endocrine 05May2009

DO NOT DISTRIBUTE - 30 -

b) Dopamine agonist

c) ADH (vasopressin)

d) Prolactin (PRL)

e) Luteinizing hormone (LH)

8) Antidiuretic hormone (ADH, vasopressin) may be indicated in which of the following

patients?

a) Diabetes mellitus type-1

b) Diabetes mellitus type-2

c) Diabetes insipidus

d) Hypertension

e) Galactorrhea

Endocrine #10 – Pharmacology: Thyroid & Antithyroid Drugs

1.1) Iodine peroxidase, which converts I- to I2, occurs in what location?

a) Thyroid gland membrane

b) Within the thyroid gland

c) Blood

d) Peripheral tissues

1.2) Amiodarone, corticosteroids, beta-blockers, and ipodate affect thyroid hormone

formation at what location?

a) Thyroid iodine transporter

b) Thyroid peroxidase enzyme

c) Thyroid T3/T4 proteolysis to blood

d) T4 to T3 conversion in peripheral tissues

e) B & C

1.3) Thioamides affect thyroid hormone formation at what location?

a) Thyroid iodine transporter

b) Thyroid peroxidase enzyme

c) Thyroid T3/T4 proteolysis to blood

d) T4 to T3 conversion in peripheral tissues

e) B & C

1.4) Iodides affect thyroid hormone formation at what location?

a) Thyroid iodine transporter

b) Thyroid peroxidase enzyme

c) Thyroid T3/T4 proteolysis to blood

d) T4 to T3 conversion in peripheral tissues

e) B & C

2) Levothyroxine (Synthroid, Levoxyl) is the preparation of choice for thyroid

replacement. Which of the following is NOT true of levothyroxine?

a) Short half-life

b) Low drug cost

c) Lack of allergenic foreign protein

d) Uniformity and stability

e) Easy lab measurement of serum levels

3) All thioamines have a simple molecular structure in common. What is the name of this

structure (shown)?

Page 31: Endocrine Pre-test self test

Endocrine 05May2009

DO NOT DISTRIBUTE - 31 -

a) Thioacetamide

b) Thionamide

c) Triiodothyronine

d) Thiocarbamide

e) Thioacetamide

4) An elderly patient is receiving levothyroxine and having their serum TSH and free

thyroxine monitored. Which of the following signs or symptoms may signify an adverse

effect to the medication?

a) Weight gain

b) Bradycardia

c) Heart palpitations

d) Cold intolerance

e) Somnolence

5) A patient with known hypothyroidism presents in a near-coma condition with edema,

bradycardia, and low core body temperature (myxedema coma). Which of the following

would be considered appropriate treatment?

a) Oral levothyroxine, 50-100mcg (low dose)

b) Oral levothyroxine, 300-400mcg (high dose)

c) IV levothyroxine, 50-100mcg (low dose)

d) IV levothyroxine, 300-400mcg (high dose)

e) IV fluids and glucose only

6) A non-pregnant patient with Graves disease is being examined for treatment options.

What is the preferred drug for this patient, which requires 12-18 months of therapy?

a) Desmopressin (DDAVP)

b) Fludrocortisone (Florinef)

c) Levothyroxine (Synthroid)

d) Methimazole (Tapazole)

e) Octreotide (Sandostatin)

Endocrine #11 – Pharmacology: Corticosteroids & Antagonists

1.1) Deficiency of the enzyme 17alpha-hydroxylase would prevent the formation of

which of the following and have what effect on blood pressure?

a) No aldosterone or cortisol, hypotension

b) No aldosterone or cortisol, hypertension

c) No cortisol or sex hormones (estradiol/testosterone), hypotension

d) No cortisol or sex hormones (estradiol/testosterone), hypertension

1.2) Deficiency of the enzyme 21-hydroxylase would prevent the formation of which of

the following and have what effect on blood pressure?

a) No aldosterone or cortisol, hypotension

b) No aldosterone or cortisol, hypertension

c) No cortisol or sex hormones (estradiol/testosterone), hypotension

d) No cortisol or sex hormones (estradiol/testosterone), hypertension

1.3) Deficiency of the enzyme 11beta-hydroxylase would prevent the formation of which

of the following and have what effect on blood pressure?

a) No aldosterone or cortisol, hypotension

b) No aldosterone or cortisol, hypertension

Page 32: Endocrine Pre-test self test

Endocrine 05May2009

DO NOT DISTRIBUTE - 32 -

c) No cortisol or sex hormones (estradiol/testosterone), hypotension

d) No cortisol or sex hormones (estradiol/testosterone), hypertension

2) Which of the following describes the hierarchy of anti-inflammatory effects for the

included drugs?

a) Hydrocortisone > Prednisone > Triamcinolone > Dexamethasone

b) Triamcinolone > Dexamethasone > Hydrocortisone > Prednisone

c) Dexamethasone > Triamcinolone > Prednisone > Hydrocortisone

d) Hydrocortisone > Triamcinolone > Dexamethasone > Prednisone

e) Dexamethasone > Hydrocortisone > Prednisone > Triamcinolone

3) Which of the following describes the hierarch of salt-retaining efficacy for the

included drugs?

a) Fludrocortisone > Hydrocortisone > Triamcinolone

b) Fludrocortisone > Triamcinolone > Hydrocortisone

c) Hydrocortisone > Fludrocortisone > Triamcinolone

d) Hydrocortisone > Triamcinolone > Fludrocortisone

e) Triamcinolone > Hydrocortisone > Fludrocortisone

4) In the fasting state, glucocorticoids stimulate phosphoenolypyruvate, carboxykinase,

and G6P. What major effect does this have?

a) Increased urine amino acids

b) Increased serum amino acids

c) Decreased gluconeogenesis

d) Increased gluconeogenesis

e) Decreased glycogen synthesis

5) Which of the following would occur in serum (blood) after injection of a short-acting

glucocorticoid?

a) Increased neutrophils and increased lymphocytes

b) Increased neutrophils and decreased lymphocytes

c) Decreased neutrophils and increased lymphocytes

d) Decreased neutrophils and decreased lymphocytes

6) A defect in 21beta-hydroxylase would prevent formation of cortisol, which leads to a

compensatory increase in ACTH. This causes the adrenal gland to become hyperplastic

and secrete large amounts of precursors. What effect would this have?

a) Hypotension

b) Hypokalemia

c) Salt wasting

d) Alopecia

e) Virilization

7) Cushing disease (ACTH-secreting pituitary adenoma) can be treated with metyrapone

(Metopirone) or which of the following?

a) Desmopressin (DDAVP)

b) Ketoconazole (Nizoral)

c) Levothyroxine (Synthroid)

d) Methimazole (Tapazole)

e) Somatotropin (Humatrope)

Page 33: Endocrine Pre-test self test

Endocrine 05May2009

DO NOT DISTRIBUTE - 33 -

8) Mifepristone (RU-486) is a progesterone antagonist with some anti-glucocorticoid

properties at higher doses. Which of the following is describes the mechanism by which

mifepristone acts with blockage of glucocorticoid receptors?

a) Stabilizes the Hsp-glucocorticoid receptor complex

b) Inhibits dissociation of the RU-486-bound glucocorticoid receptor from the

Hsp chaperone proteins

c) Alters the interaction of the glucocorticoid receptor with coregulators, favoring

the formation of a transcriptionally inactive complex

d) All of the above

Endocrine #12 – Pharmacology: Gonadal Hormones & Inhibitors

1.1) Which of the following does NOT peak at day 14 of a 28-day menstrual cycle?

a) LH

b) FSH

c) Estradiol

d) Progesterone

1.2) At which of the following days during a 28-day menstrual cycle would endometrial

lining be the thickest?

a) 7

b) 14

c) 21

d) 28

2) Which of the following steroid reactions is NOT correct?

a) Androstenedione can convert to testosterone without an enzyme

b) Testosterone requires aromatase to create estradiol

c) Estradiol requires aromatase to create estrone

d) Androstenedione requires aromatase to create estrone

e) Testosterone can convert to androstenedione without an enzyme

3) What effect does FSH have on the follicle of the mature ovary?

a) Stimulates growth of ovarian follicles and estrogen secretion

b) Stimulates growth of ovarian follicles and progesterone secretion

c) Stimulates recession of ovarian follicles and estrogen secretion

d) Stimulates recession of ovarian follicles and progesterone secretion

4) Estrogen (estradiol) receptors are found in the nucleus in what stable conformation?

a) Bound to estrogen response elements (EREs)

b) Bound to heat shock protein (Hsp90)

c) As a dimer (estradiol-estradiol)

d) Bound to estrone

e) In an aromatic ring-form pattern

5) According to the Women’s Health Initiative (WHI), estrogen treatment or hormone

replacement therapy (HRT) in peri- and post-menopausal women can increase the risk of

cardiovascular disease as well as which of the following?

a) Increased risk of Alzheimer disease

b) Negative effect on circulating lipids

c) Increased risk of breast cancer

d) Increased risk of colon cancer

Page 34: Endocrine Pre-test self test

Endocrine 05May2009

DO NOT DISTRIBUTE - 34 -

e) Negative effect on circulating lipoproteins

6) What is the claimed advantage of third-generation synthetic progestins (19-not, 13-

ethyl), including desogestrel, gestodene, and norgestimate, when compared with older

synthetic progestins?

a) Less effective gonadotropin inhibitors

b) Less estrogenic activity

c) More estrogenic activity

d) Less androgenic activity

e) More androgenic activity

7) Which of the following is an adverse effect of oral contraceptives (estrogenic

compounds), which has increased risk in smokers over aged 35?

a) Bleeding disorders

b) Small cell lung cancer

c) Duodenal ulcers

d) Rheumatic fever

e) Venous embolism

8) Which of the following post-coital contraceptives is considered abortifacient?

a) Conjugate estrogen

b) Ethinyl estradiol

c) Mifepristone

d) Diethylstilbestrol

e) Norgestrel

9.1) What is the role of LH on the testis?

a) Promote sperm maturation

b) Promote testosterone synthesis

c) Promote testosterone secretion

d) A & B

e) B & C

9.2) What is the role of FSH on the testis?

a) Promote sperm maturation

b) Promote testosterone synthesis

c) Promote testosterone secretion

d) A & B

e) B & C

10) In the skin, prostate, and seminal vesicles, testosterone is converted to which of the

following?

a) DHEA

b) DHT

c) ACTH

d) Cortisol

e) Estradiol

11) Which of the following is NOT correct regarding the dosing regimen of androgens in

cases of pubertal hypogonadism?

a) Long-acting agents such as testosterone enanthate or cypionate are used

b) Doses are 50mg IM injection

Page 35: Endocrine Pre-test self test

Endocrine 05May2009

DO NOT DISTRIBUTE - 35 -

c) Initial doses are every 4-weeks, then every 3-weeks, then every 2-weeks until

maturation is complete

d) Adult doses are 200mg at 4-week intervals

12.1) Finasteride (Proscar) inhibits the production of DHT by inhibiting what enzyme?

a) 17alpha-hydroxylase

b) Aromatase

c) 11beta-hydroxylase

d) 5alpha-reductase

e) 21-hydroxylase

12.2) Flutamide (Eulexin) has which of the following effects?

a) Agonist at the androgen receptor

b) Competitive antagonist at the androgen receptor

c) Inhibitor of 5alpha-reductase

d) Activator of 5alpha-reductase

Endocrine #13 – Pharmacology: Pancreatic Hormones & Antidiabetic Drugs

1.1) Which of the following is found in beta cells of the pancreas and regulates insulin

release as well as other aspects of glucose homeostasis?

a) GLUT1

b) GLUT2

c) GLUT3

d) GLUT4

e) GLUT5

1.2) Which of the following is found in muscle and adipose tissue and functions in

insulin-mediated uptake of glucose?

a) GLUT1

b) GLUT2

c) GLUT3

d) GLUT4

e) GLUT5

2) Which of the following describes the hierarchy of time to peak action for the included

insulin formations?

a) Lispro insulin > Regular insulin > NPH > Glargine insulin

b) Regular insulin > Lispro insulin > NPH > Glargine insulin

c) NPH > Glargine insulin > Lispro insulin > Regular insulin

d) Regular insulin > Glargine insulin > Lispro insulin > NPH

e) Glargine insulin > NPH > Regular insulin > Lispro insulin

3) How do sulfonylurea drugs affect insulin release?

a) Blocks GLUT2

b) Depolarizes (closes) K+ channels

c) Polarizes (opens) K+ channels

d) Depolarizes (opens) Ca++ channels

e) Polarizes (closes) Ca++ channels

f) Blocks insulin exocytosis

4) Why are children under the age of seven exempt from the American Diabetes

Association’s Benefits of Tight Glycemic Control of Diabetes guidelines?

Page 36: Endocrine Pre-test self test

Endocrine 05May2009

DO NOT DISTRIBUTE - 36 -

a) Risk of thromboembolic events

b) Risk of brain damage due to hypoglycemia

c) Risk of neuropathy due to hyperglycemia

d) Risk of renal failure due to glycemic fluctuations

e) Risk of permanent bone mass loss

5) Which of the following is NOT an alternative to insulin syringe injections currently

available for Type-1 diabetics?

a) Subcutaneous insulin infusion devices

b) Portable pen injectors

c) Ocular insulin drops

d) Inhaled insulin

6) Risk is most reduced in what system for patients who adhere to the guidelines for

Benefits of Tight Glycemic Control of Diabetes?

a) Neurologic

b) Gastrointestinal

c) Renal

d) Reproductive

e) Cardiovascular

7) Which of the following is NOT a symptom of hypoglycemia?

a) Difficulty speaking

b) Tachycardia

c) Convulsions

d) Pinpoint pupils (miosis)

e) Headache

8) When are alpha-glucosidase inhibitors indicated?

a) Initial therapy for Type-1 diabetics

b) Adjunctive therapy for Type-1 diabetics

c) Initial therapy for Type-2 diabetics

d) Adjunctive therapy for Type-2 diabetics

e) Never due to their side-effect profile

9) Which of the following describes the hierarchy of maximal duration of action for the

included sulfonylureas?

a) Tolbutamide > Glimepiride > Chlorpropamide

b) Glimepiride > Chlorpropamide > Tolbutamide

c) Chlorpropamide > Glimepiride > Tolbutamide

d) Chlorpropamide > Tolbutamide > Glimepiride

e) Tolbutamide > Chlorpropamide > Glimepiride

10) Which of the following is the safest sulfonylurea for elderly diabetics and why?

a) Chlorpropamide due to its long half-life

b) Chlorpropamide due to its short half-life

c) Tolbutamide due to its long half-life

d) Tolbutamide due to its short half-life

e) Glimepiride due to its long half-life

11) Which of the following is NOT a typical reason for secondary failure (failure to

maintain a good response to sulfonylurea over the long term) in patients on sulfonylurea

therapy?

Page 37: Endocrine Pre-test self test

Endocrine 05May2009

DO NOT DISTRIBUTE - 37 -

a) Reduction in physical activity

b) Decline in lean body mass

c) Decline in fat in chronic type-2 diabetes

d) Decrease in B-cell mass

12) Why should meglitinides (repaglinide) be avoided as adjunctive treatment in patients

taking a sulfonylurea drug?

a) Their binding sites overlap

b) They bind to each other forming a non-soluble product

c) They bind to each other, which inactivates the sulfonylurea agent

d) They bind to each other, which inactivates the meglitinide agent

e) They can be used safely in combination

13) Which of the following is NOT a postulated mechanism of action for biguanides?

a) Reduced hepatic and renal gluconeogenesis

b) Inhibition of pancreatic beta cells

c) Slowing of glucose absorption from GI tract with increased glucose to lactate

conversion by enterocytes

d) Direct stimulation of glycolysis in tissues with increased glucose removal from

blood

e) Reduction of plasma glucagon levels

14) Thiazolidinediones (Tzds) bind to peroxisome proliferator-activated receptor-gamma

(PPAR-g) receptors in muscle, fat, and liver. Which of the following is NOT a receptor

involved in gene activation by Tzds?

a) Lipid metabolism

b) Insulin signal tranduction

c) Adipose tissue differentiation

d) Glucose metabolism

e) Glucagon signal tranduction

15) Which of the following is NOT a postulated mechanism of action for exenatide?

a) Central loss of appetite

b) Potentiating of glucose-mediated insulin secretion

c) Enhanced gastric emptying

d) Suppression of postprandial glucagon release

Endocrine #14 – Pharmacology: Agents That Affect Bone Mineral Homeostasis

1) Which of the following reactions is inhibitory?

a) Serum calcium/phosphate to bone via vitamin D

b) Serum calcium/phosphate to bone via PTH

c) Bone to serum calcium/phosphate via vitamin D

d) Bone to serum calcium/phosphate via PTH

e) Bone to serum calcium/phosphate via calcitonin

2) What affect do bisphosphonates have on farnesyl-pyrophosphate synthase?

a) Inhibitory leading to inhibited osteoclast function

b) Inhibitory leading to enhanced osteoclast function

c) Stimulatory leading to inhibited osteoclast function

d) Stimulatory leading to enhanced osteoclast function

Page 38: Endocrine Pre-test self test

Endocrine 05May2009

DO NOT DISTRIBUTE - 38 -

3) Which of the following is a selective estrogen receptor modulator (SERM) approved

for the treatment of osteoporosis?

a) Octreotide (Sandostatin)

b) Alendronate (Fosemax)

c) Raloxifene (Evista)

d) Ergocalciferol (Calciferol)

e) Calcitonin (Calcimar)

4) What advantage does teriparatide have over fluoride in the treatment of osteoporosis?

a) Reduced incidence of hypocalcemia

b) Reduced incidence of hypercalcemia

c) Reduced incidence of CNS symptoms

d) Reduced incidence of bone fractures

e) Reduced incidence of prolonged bleeding

5) Which of the following is a bisphosphonate approved for osteoporosis?

a) Octreotide (Sandostatin)

b) Alendronate (Fosemax)

c) Raloxifene (Evista)

d) Ergocalciferol (Calciferol)

e) Calcitonin (Calcimar)

6) Bisphosphonates may be administered via IV or orally. If administered orally, which

of the following protocols should be followed?

a) Take on an empty stomach without any fluids

b) Take on an empty stomach with water and stay upright

c) Take with food and avoid staying still for prolonged periods

d) Take with milk only or a lactose-like liquid for vegan patients

e) Take with a full glass of red wine or shot of 80-proof liquor

7) Which of the following drugs is used for osteoporosis and stimulates new bone

formation, rather than inhibiting bone resorption?

a) Reveromycin A

b) Indomethacin

c) Diphenylhydantoin

d) Teriparatide

e) Alendronate

Endocrine #15 – Pathology

1) Which of the following is more characteristic of Type-2 diabetes mellitus compared

with Type-1 pancreatic morphologic changes?

a) Leukocyte infiltration of the islets

b) Great reduction in islet cell number and size

c) Amyloid replacement of islet cells

d) Beta cell degranulation

2.1) What is the number one complication for patients with diabetes mellitus?

a) Neuropathy

b) Foot gangrene

c) Atherosclerosis

d) Glomerulosclerosis

Page 39: Endocrine Pre-test self test

Endocrine 05May2009

DO NOT DISTRIBUTE - 39 -

e) Hyperlipidemia

2.2) What is the most common cause of death in diabetics?

a) Myocardial infarction

b) Stroke

c) Renal failure

d) Kimmelstein-Wilson lesions

e) Hypertensive crisis

2.3) Which of the following complications of diabetes is caused by hypertension?

a) Diabetic macrovascular disease

b) Hyaline arteriosclerosis

c) Diabetic microangiopathy

d) Diabetic nephropathy

e) Diffuse mesangial sclerosis

f) Nodular glomerulosclerosis

2.4) Which of the following involves PAS-positive lesions which may trap mesangial

cells (Kimmelstein-Wilson lesions) and if present is essentially pathognomonic for

diabetes?

a) Diabetic macrovascular disease

b) Hyaline arteriosclerosis

c) Diabetic microangiopathy

d) Diabetic nephropathy

e) Diffuse mesangial sclerosis

f) Nodular glomerulosclerosis

2.5) Which of the following complications of diabetes is responsible for the disease-

defining neuropathy, nephropathy, and retinopathy?

a) Diabetic macrovascular disease

b) Hyaline arteriosclerosis

c) Diabetic microangiopathy

d) Diabetic nephropathy

e) Diffuse mesangial sclerosis

f) Nodular glomerulosclerosis

2.6) Which of the following is NOT characteristic of Type-1 diabetes mellitus?

a) Ketoacidosis

b) Polyuria

c) Polydipsia

d) Polyphagia

e) Obesity

2.7) Which of the following describes the type of coma seen in dehydrated Type-2

diabetes mellitus patients?

a) Hypo-osmolar, non-ketonic

b) Hyper-osmolar, non-ketonic

c) Hypo-osmolar, ketonic

d) Hyper-osmolar, ketonic

2.8) What is the first sign of diabetic nephropathy in Type-2 diabetics?

a) Failure to concentrate urine

b) Subtle amounts of albumin in urine

Page 40: Endocrine Pre-test self test

Endocrine 05May2009

DO NOT DISTRIBUTE - 40 -

c) Dark, strong smelling urine

d) High LDL and TG blood levels

e) Low HDL blood levels

3.1) A patient presents with confusion and stupor after running a marathon. Testing

shows a blood glucose level below 50mg/dL. The patient’s friend says these symptoms

have occurred before during marathon training. Pancreatic imaging shows small,

encapsulated, solitary tumors. Which of the following is most likely?

a) Beta-cell tumor (insulinoma)

b) Zollinger-Ellison syndrome

c) Alpha-cell tumor (glucagonoma)

d) Delta-cell tumor (somatostatinoma)

e) VIPoma

3.2) A patient presents with steatorrhea, hypochlorhydria, cholelithiasis, and diabetes

mellitus. Which of the following is most likely?

a) Beta-cell tumor (insulinoma)

b) Zollinger-Ellison syndrome

c) Alpha-cell tumor (glucagonoma)

d) Delta-cell tumor (somatostatinoma)

e) VIPoma

3.3) A patient with a history of multiple endocrine neoplasia type 1 (MEN-1, Wermer

syndrome) presents with peptic ulcers and diarrhea. Testing shows tumor metastasis.

Which of the following is most likely?

a) Beta-cell tumor (insulinoma)

b) Zollinger-Ellison syndrome

c) Alpha-cell tumor (glucagonoma)

d) Delta-cell tumor (somatostatinoma)

e) VIPoma

3.4) A post-menopausal woman presents with a skin rash. Testing shows anemia and a

glucose level suggestive of diabetes mellitus. Which of the following is most likely?

a) Beta-cell tumor (insulinoma)

b) Zollinger-Ellison syndrome

c) Alpha-cell tumor (glucagonoma)

d) Delta-cell tumor (somatostatinoma)

e) VIPoma

3.5) Which of the following is associated with watery diarrhea, hypokalemia, and

achlorhydria (WDHA syndrome)?

a) Beta-cell tumor (insulinoma)

b) Zollinger-Ellison syndrome

c) Alpha-cell tumor (glucagonoma)

d) Delta-cell tumor (somatostatinoma)

e) VIPoma

4.1) A young woman is being treated in the emergency department for psychosis and is

given haloperidol and reserpine. After developing galactorrhea, the patient admits to not

having her period in some time (amenorrhea). A pregnancy test is negative. After a CT

scan of the brain returns, the clinician opts to avoid invasive prodecures but speculates

she would have found dystrophic calcification. Which of the following is most likely?

Page 41: Endocrine Pre-test self test

Endocrine 05May2009

DO NOT DISTRIBUTE - 41 -

a) Somatotrophic adenoma

b) Prolactinoma

c) Corticotrophic adenoma

d) Empty sella syndrome

e) Sheehan syndrome

4.2) A patient presents with a history of Cushing syndrome which was resolved after

removal of the adrenal glands. CT scan shows an anterior lobe pituitary adenoma that has

grown since the operation (Nelsons syndrome). A post-mortem biopsy of the tumor

shows densely granulated and PAS-positive staining. Which of the following is most

likely?

a) Somatotrophic adenoma

b) Prolactinoma

c) Corticotrophic adenoma

d) Empty sella syndrome

e) Sheehan syndrome

4.3) An adult presents with acromegaly and a broad face. Testing shows high levels of

serum GH and IGF-1. An oral load of glucose is administered, but serum GH levels are

still high. A CT scan shows a large pituitary adenoma of the anterior lobe. What is the

mechanism by which this tumor arose?

a) Defect in p53 gene causing inability to force cell apoptosis

b) Defect in GTPase of the alpha-subunit of G-protein (Gs)

c) Defect in bcl-2 and bak genes

d) Defect in proto-oncogenes bcl-2 and c-myc

e) Gain on function mutation resulting in cAMP overproduction

5) During parturition, a woman develops obstetrical hemorrhage, which is controlled by

the hospital staff. Shortly after birth, the woman develops hypotension, starts losing her

pubic hair, and is unable to breast feed. CT scanning of the head shows likely ischemic

necrosis in the sella turcica. Which of the following is most likely?

a) Somatotrophic adenoma

b) Prolactinoma

c) Corticotrophic adenoma

d) Empty sella syndrome

e) Sheehan syndrome

6) An obese woman with hypertension presents with complains of visual defects. Testing

shows hyperprolactinemia. CT scan shows herniation of the arachnoid matter and

cerebrospinal fluid into the sella turcica. Which of the following is most likely?

a) Somatotrophic adenoma

b) Craniopharyngioma

c) Corticotrophic adenoma

d) Empty sella syndrome

e) Sheehan syndrome

7) A child presents with poor peripheral vision. A CT scan shows a cystic, multi-

lobulated tumor above the sella turcica. Surgical removal is performed and the tumor is

sent to pathology. Testing shows dystrophic calcification and cysts lined with squamous

epithelium. The cysts are filled with a yellow, viscous fluid that is rich in cholesterol

Page 42: Endocrine Pre-test self test

Endocrine 05May2009

DO NOT DISTRIBUTE - 42 -

crystals. The tumor likely developed from vestigial remnants of Rathkes pouch. Which of

the following is most likely?

a) Somatotrophic adenoma

b) Craniopharyngioma

c) Corticotrophic adenoma

d) Empty sella syndrome

e) Sheehan syndrome

8) An ICU patient with recent head trauma is found to be creating excess urine. Testing

of the urine shows a low specific gravity. Serum testing shows elevated sodium. The

patient complains of being very thirsty. ADH deficiency is suspected. Which of the

following is most likely?

a) Central diabetes insipidus

b) Nephrogenic diabetes insipidus

c) Type-1 diabetes mellitus

d) Type-2 diabetes mellitus

e) Syndrome of inappropriate ADH secretion (SIADH)

9) A patient presents with shortness of breath. Testing reveals high ADH levels. Chest x-

ray reveals small cell carcinoma of the lung. If SIADH is suspected, which of the

following would be expected on labs and during physical exam?

a) Hypernatremia and edema

b) Hyponatremia and edema

c) Hypernatremia without edema

d) Hyponatremia without edema

10) Which of the following is a sign of hypothyroidism?

a) Lateral eyebrow hair loss

b) Patient always feels hot (hyperpyrexia)

c) Excessive perspiration

d) Palpitations, tachycardia

e) High systolic pressure with normal diastolic pressure (high pulse pressure)

11) Which of the following is a sign of hyperthyroidism (e.g. thyrotoxicosis)?

a) Retardation and deficient growth (cretinism)

b) Puffy eyelids, narrow palpebral fissure, protruded tongue

c) Fine-amplitude hand tremors

d) Cold intolerance and scaly dermatitis

e) Deftness, night blindness, somnolence, dulled reflexes

12) Which of the following would NOT cause symptoms similar to hypothyroidism?

a) Thyroid hormone resistance syndrome

b) Autoimmune (Hashimoto) thyroiditis

c) Use of lithium, iodides, or p-aminosalicyclic acid

d) Excessive iodine

e) Pituitary failure

13) A middle-aged woman from a third world country presents complaints of feeling

tired. History reveals Sjögren syndrome. Physical exam reveals an enlarged, non-nodular

thyroid. Labs reveal euthyroid. Biopsy shows inflammatory destruction of thyroid

follicles and Hürthle cells (large eosinophilic cells showing oxyphilic metaplasia of

thyroid follicular cells). Which of the following is most likely?

Page 43: Endocrine Pre-test self test

Endocrine 05May2009

DO NOT DISTRIBUTE - 43 -

a) Subacute (DeQuervain) thyroiditis

b) Primary hyperthyroidism

c) Graves disease

d) Lymphocytic (Hashimoto) thyroiditis

e) Primary hypothyroidism

14) A patient presents with jaw and neck pain with swallowing. History reveals a recent

influenza infection. Exam reveals hoarseness and an exquisitely tender and enlarged

thyroid. A 131I-uptake test shows decreased uptake of iodide diffusely. Although a

biopsy is not performed, granulomatous inflammation with multinucleated giant cells

would be expected as well as microabscesses with neutrophilic infiltration. Which of the

following is most likely?

a) Subacute (DeQuervain) thyroiditis

b) Primary hyperthyroidism

c) Graves disease

d) Lymphocytic (Hashimoto) thyroiditis

e) Primary hypothyroidism

15) A middle-aged female presents with agitation and hyperpyrexia. On ocular exam, the

patient appears to be wide-eyed and extremely alert. Labs reveal HLA-B8 and DR3. IgG

autoantibodies to the TSH receptor is suspected. Although a biopsy is not performed, a

scalloped “moth-eaten” thyroid appearance would be expected. Which of the following is

most likely?

a) Subacute (DeQuervain) thyroiditis

b) Primary hyperthyroidism

c) Graves disease

d) Lymphocytic (Hashimoto) thyroiditis

e) Primary hypothyroidism

16) A patient from central Asia presents with an enormous, multinodular neck mass.

History reveals a diet rich in cabbage, turnips, and Brussel sprouts. Biopsy reveals the

plunging goiter is very rick in colloid material. Which of the following demographics is

most likely for this patient?

a) Young boy

b) Young girl

c) Elderly man

d) Elderly woman

e) Pregnant woman

17) What is the hallmark of a follicular adenoma?

a) Painless neck mass with dysphagia

b) Hürthle cells with eosinophilic granular cytoplasm

c) Well-formed capsule around the tumor

d) Multinodular with a capsule that is penetrated by tissue

e) Overproduction of cAMP with signs of malignancy

18.1) Which of the following carcinomas of the thyroid gland is derived from

parafollicular or C cells and has a strong association with MEN-IIa and IIb?

a) Papillary carcinoma

b) Follicular carcinoma

c) Medullary carcinoma

Page 44: Endocrine Pre-test self test

Endocrine 05May2009

DO NOT DISTRIBUTE - 44 -

d) Anaplastic carcinoma

18.2) A 30-year-old woman presents with complaints of a neck lump. Physical exam

reveals a mass in the cervical lymph node. A solitary thyroid nodule is found and is freely

moveable. Resection of the mass reveals papillae (stalks covered by cuboidal epithelium)

and psammoma bodies. Microscopy shows empty cells and ground glass or Orphan

Annie eye nuclei. Which of the following is most likely?

a) Papillary carcinoma

b) Follicular carcinoma

c) Medullary carcinoma

d) Anaplastic carcinoma

18.3) A 65-year-old patient presents with a history of multinodular goiter. Testing reveals

a highly anaplastic undifferentiated neoplasm with giant cells. What is the prognosis for

this patient?

a) 0% rate of mortality

b) 25% rate of mortality

c) 50% rate of mortality

d) 75% rate of mortality

e) 100% rate of mortality

18.4) A 50-year-old woman presents with a thyroid carcinoma that has traveled

hematogenously to the bones of the shoulder, pelvis, sternum, and skull. Which of the

following is most likely?

a) Papillary carcinoma

b) Follicular carcinoma

c) Medullary carcinoma

d) Anaplastic carcinoma

19.1) Which of the following is the most common cause of hyperparathyroidism?

a) MEN-1 inactivation on chromosome 11

b) MEN-2 tyrosine kinase receptor mutation

c) Familial hypocalcinuric hypercalcemia (FHH) due to CASR gene mutation

d) Parathyroid adenoma due to relocation of PRAD1 proto-oncogene, over-

expressing cyclin D1 protein

19.2) Which of the following affects all four parathyroid glands and results in water-clear

cells?

a) Parathyroid adenoma

b) Parathyroid hyperplasia

c) Parathyroid carcinoma

19.3) A 45-year-old woman presents with bone pain, abdominal pain, and depression.

Testing reveals nephrolithiasis and elevated PTH levels. Which of the following is most

likely?

a) Hypercalcemia of malignancy

b) Vitamin D toxicity

c) Thiazide diuretic use

d) Hyperparathyroidism

e) Granulomatous disease

20) A hospitalized patient is found to have chronic depression of serum calcium levels.

Parathyroid gland analysis reveals hyperplasia with increased chief cells and water-clear

Page 45: Endocrine Pre-test self test

Endocrine 05May2009

DO NOT DISTRIBUTE - 45 -

cells. Further testing reveals bone changes and metastatic calcification (calciphylaxis).

What is the most common cause of this patient’s secondary hyperparathyroidism?

a) Vitamin D deficiency

b) Inadequate intake of calcium

c) Renal failure

d) Steatorrhea

21) A patient with DiGeorge syndrome undergoes surgery to remove lymph nodes in the

neck. Long after the surgery, the patient develops emotional instability and Parkinson-

like movements. Physical exam reveals papilledema and cataracts. Testing reveals

prolonged QT interval and calcification of the basal ganglion. Tapping on the facial nerve

induces ipsilateral muscle contraction (Chvostek sign). Blood pressure cuff occlusion

reveals carpal spasm (Trousseau sign), which disappears as soon as the cuff is removed.

Which of the following is most likely?

a) Hyperthyroidism

b) Hypothyroidism

c) Hyperparathyroidism

d) Hypoparathyroidism

e) Pseudohypoparathyroidism

f) Pseudopseudohypoparathyroidism

22.1) Pseudohypoparathyroidism is due to end-organ resistance to the actions of PTH,

thus PTH levels may be normal or elevated. This is due to G,alpha-mediated pathway

expression in the kidney of GNAS1 gene. Which of the following describes

pseudohypoparathyroidism type-1A? AHO = Albright hereditary osteodysrophy

a) AHO with multi-hormone resistance and maternal allele mutation

b) AHO with multi-hormone resistance and paternal allele mutation

c) AHO without multi-hormone resistance and maternal allele mutation

d) AHO without multi-hormone resistance and paternal allele mutation

22.2) Which of the following describes pseudopseudohypoparathyroidism?

a) AHO with multi-hormone resistance and maternal allele mutation

b) AHO with multi-hormone resistance and paternal allele mutation

c) AHO without multi-hormone resistance and maternal allele mutation

d) AHO without multi-hormone resistance and paternal allele mutation

23.1) Which of the following is the most common cause of Cushing syndrome?

a) Iatrogenic (exogenous corticosteroid administration)

b) Adrenal (hypersecreting cortisol adenoma)

c) Paraneoplastic (ACTH secretion from small cell lung cancer)

d) Pituitary (ACTH anterior pituitary tumor)

23.2) Which of the following forms of Cushing syndrome would have high serum cortisol

and low ACTH, and is also called “independent Cushing syndrome”?

a) Iatrogenic (exogenous corticosteroid administration)

b) Adrenal (hypersecreting cortisol adenoma)

c) Paraneoplastic (ACTH secretion from small cell lung cancer)

d) Pituitary (ACTH anterior pituitary tumor)

23.3) Which of the following forms of Cushing syndrome would have high serum ACTH

and cortical hyperplasia?

a) Iatrogenic (exogenous corticosteroid administration)

Page 46: Endocrine Pre-test self test

Endocrine 05May2009

DO NOT DISTRIBUTE - 46 -

b) Adrenal (hypersecreting cortisol adenoma)

c) Paraneoplastic (ACTH secretion from small cell lung cancer)

d) Pituitary (ACTH anterior pituitary tumor)

23.4) Which of the following would occur in Cushing syndrome caused by small cell

cancer of the lung?

a) Increased serum ACTH and cortisol

b) Decreased serum ACTH and cortisol

c) Increased ACTH and decreased cortisol

d) Decreased ACTH and increased cortisol

e) No change in serum ACTH or corisol

23.5) A patient presents with central trunk obesity, a “buffalo hump,” moon facies,

muscle wasting, hirsutism, and hypertension. A 24-hours urine free cortisol level suggests

Cushing syndrome. Dexamethasone is given to the patient and ACTH levels are

suppressed. Which of the following forms is most likely?

a) Iatrogenic (exogenous corticosteroid administration)

b) Adrenal (hypersecreting cortisol adenoma)

c) Paraneoplastic (ACTH secretion from small cell lung cancer)

d) Pituitary (ACTH anterior pituitary tumor)

24) After finding high levels of serum aldosterone and low levels of serum renin,

hyperaldosteronism is suspected. Dexamethasone is found to suppress aldosterone

production, so an adrenal CT is performed, which finds an adenoma (Conn syndrome).

Excision reveals bright-yellow, lipid-laden cortical cells with spironolactone inclusion

bodies. Which of the following would also be expected in this patient?

a) Hypotension and hypokalemia

b) Hypotension and hyperkalemia

c) Hypertension and hypokalemia

d) Hypertension and hyperkalemia

25) A female newborn is found to have increased androgens, decreased cortisol, and

increased ACTH secretion. She develops virilization (e.g. hirsutism) and bilateral

hyperplasia of the adrenal glands (congenital hyperplasia). Which of the following

enzymes is most likely deficient?

a) 17alpha-hydroxylase

b) Aromatase

c) 11beta-hydroxylase

d) 5alpha-reductase

e) 21-hydroxylase

26) Which of the following would NOT cause adrenocortical insufficiency?

a) Acute hemorrhagic necrosis (Waterhouse-Friderichsen syndrome)

b) Congenital adrenal hyperplasia

c) Adrenoleukodystrophy (ALD gene on Xq28)

d) Autoimmune poly-endocrinopathy syndrome

e) AIDS, infection, fungi, tuberculosis

27) A patient presents with photophobia, fatigue, and neck pain. Testing and exam

reveals severe hypotension, purpura, and disseminated intravascular coagulation (DIC).

Antibiotics are started immediately in hopes of preventing massive bilateral adrenal

Page 47: Endocrine Pre-test self test

Endocrine 05May2009

DO NOT DISTRIBUTE - 47 -

hemorrhage (Waterhouse-Friderichsen syndrome). What bacterium is most likely

responsible?

a) Streptococcus agalactiae

b) Mycobacterium tuberculosis

c) Neisseria meningitidis

d) Escherichia coli

e) Listeria monocytogenes

28) A patient presents with progressive weakness and easy fatigability. They complain of

GI disturbances and have hyperpigmentation. Labs show hypokalemia, hyponatremia,

and volume depletion. Exam reveals hypotension. Further testing reveals shrunken

adrenal glands. The clinician suspects that the adrenal glands have lost over 90% of their

function. Which of the following is most likely?

a) Primary adrenocortical insufficiency (Addison disease)

b) Autoimmune poly-endocrinopathy syndrome

c) Adrenoleukodystrophy (ALD gene on Xq28)

d) X-linked adrenal hypoplasia (DAX-1 gene on Xp21)

e) Acute hemorrhagic necrosis (Waterhouse-Friderichsen syndrome)

29) Which of the following would NOT be seen in secondary adrenocortical

insufficiency, as aldosterone synthesis is normal?

a) Decreased ACTH levels

b) Deficiency in cortisol output

c) Deficiency in androgen output

d) Hyponatremia, hypokalemia, hyperpigmentation

e) Exogenous ACTH elevates plasma cortisol levels

30) Which of the following does NOT describes an adrenal cortical carcinoma?

a) Associated with virilism

b) Can be caused by LiFraumeni syndrome or Beckwith-Wiedemann syndrome

c) Involves a large and invasive area of necrosis and hemorrhage

d) Hyperplastic, well circumscribed, yellow (lipids), nodular lesion

e) Commonly invades the adrenal vein and inferior vena cava

31) A patient presents with palpitations and excessive sweating. Testing shows

orthostatic hypertension. History reveals dizziness when lifting weights at the gym and

sometimes (paradoxically) while defecating. Further testing shows large balls of

neoplastic chromaffin cells in the adrenal gland (zellballen). Urinalysis reveals excess

vanillylmandelic acid (VMA) due to epinephrine and norepinephrine breakdown. Where

is the tumor most likely located?

a) Adrenal cortex, zona glomerulosa

b) Adrenal cortex, zona fasciculata

c) Adrenal cortex, zona reticularis

d) Adrenal medulla

e) Extra-adrenal

32) Where is a common location to find an extramedullary paraganglioma?

a) Sternoclavicular joint

b) Carotid body

c) Cisterna chyle

d) Ligaments of the knee

Page 48: Endocrine Pre-test self test

Endocrine 05May2009

DO NOT DISTRIBUTE - 48 -

e) GI tract at splenic flexure

33) Which of the following patients is most likely to develop thymic hyperplasia, which

is lymphoid follicles containing mostly B cells?

a) Hashimoto thyroiditis

b) Sjögren disease

c) Multiple sclerosis

d) Myasthenia gravis

e) Pregnant diabetic

34) Thymomas typically present either due to impingement of the anterior superior

mediastinum or due to:

a) Hashimoto thyroiditis

b) Sjögren disease

c) Multiple sclerosis

d) Myasthenia gravis

e) Pregnant diabetic

35) Which of the following is NOT associated with thymoma?

a) Graves disease

b) Cushing syndrome

c) Pernicious anemia

d) Megaloblastic anemia

e) Dermatomyositis-polymyositis

f) Acquired hypogammaglobulinemia

36) An adult presents with visual disturbances, mental deterioration, and dementia-like

behavior. Head CT shows a tumor near the third ventricle that is nearly impeding the

aqueduct of Sylvius. Treatment is started to prevent internal hydrocephalus. Which of the

following is most likely?

a) Medulloblastoma

b) Retinoblastoma

c) Thymoma

d) Pineoblastoma

e) Pineocytoma

37.1) Which of the following areas is NOT affected by MEN-1 (Wermer) syndrome?

a) Pituitary

b) Parathyroid

c) Thyroid

d) Pancreatic islets

e) Adrenal gland

37.2) MEN-2A (Sipple syndrome) and MEN-2B (MEN-III) most often affect what areas?

a) Pituitary and parathyroid

b) Parathyroid and pancreatic islets

c) Pancreatic islets and adrenal

d) Adrenal and thyroid

e) Thyroid and pituitary

37.3) MEN-2B does not induce hyperparathyroidism like MEN-2A does. However, both

types involve mutations in what gene?

a) p53

Page 49: Endocrine Pre-test self test

Endocrine 05May2009

DO NOT DISTRIBUTE - 49 -

b) MEN-1

c) RET

d) Bcl-2

e) c-myc

Endocrine #16 – Clinical: Hypothalamic Pituitary Disorders

1) Which of the following is NOT a common cause of hypopituitarism?

a) Radiotherapy

b) Autoimmunity

c) Pituitary tumors

d) Pituitary operations

2) Which of the following is NOT a symptom of gonadotropin deficiency in women?

a) Amenorrhea

b) Loss of libido

c) Dyspareunia

d) Hirsutism

e) Vaginal dryness

3) Which of the following is NOT a symptom of gonadotropin deficiency in men?

a) Loss of libido

b) Erectile dysfunction

c) Gynecomastia

d) Infertility

e) Hypertrophy of the testes

4) Which of the following would NOT be low (or inappropriately normal) in

hypopituitarism?

a) TSH

b) FSH

c) LH

d) Estradiol

e) Testosterone

5) Which of the following hormone abnormalities would be expected in supporting a

diagnosis of central hypothyroidism?

a) Increased serum FT4 and increased serum TSH

b) Increased serum FT4 and decreased serum TSH

c) Decreased serum FT4 and increased serum TSH

d) Decreased serum FT4 and decreased serum TSH

6) Which of the following is a recently developed test to stimulate growth hormone

secretion?

a) Tryptophan plus GHRH

b) Arginine plus GHRH

c) Alanine plus GHRH

d) Aspartate plus GHRH

e) Glutamine plus GHRH

7) What hormone replacement is essential in ACTH deficiency?

a) Glucocorticoid replacement (Hydrocortisone)

b) Levothroxine sodium (Synthroid)

Page 50: Endocrine Pre-test self test

Endocrine 05May2009

DO NOT DISTRIBUTE - 50 -

c) Testosterone (IM or transdermal patch)

d) Somatotropin (Humatrope)

e) Oxytocin (Pitocin)

8) What is the drug of choice for TSH deficiency?

a) Glucocorticoid replacement (Hydrocortisone)

b) Levothroxine sodium (Synthroid)

c) Testosterone (IM or transdermal patch)

d) Somatotropin (Humatrope)

e) Oxytocin (Pitocin)

9) For women who do not desire fertility, what is the hormone replacement therapy used

for gonadotropin deficiency?

a) Estrogen therapy

b) Progesterone therapy

c) Gonadotropin therapy

d) A & B

e) B &C

10) What is the hormone replacement therapy used in males with gonadotropin

deficiency?

a) Estrogen therapy

b) Progesterone therapy

c) Testosterone therapy

d) FSH therapy

e) LH therapy

Endocrine #17 – Clinical: Pituitary Tumors

1) What tumor size distinguishes a microadenoma from a macroadenoma?

a) 1cm

b) 2cm

c) 3cm

d) 4cm

e) 5cm

2) Which of the following is NOT a key question to ask when evaluating a pituitary

tumor?

a) Is the tumor causing a local mass effect?

b) Is hypopituitarism present?

c) Is there evidence of hormone excess?

d) Is the patient pregnant and at risk for Sheehan syndrome?

3) Which of the following is NOT a neurological defect that can occur with a pituitary

tumor?

a) Visual field and acuity defects (superior tumor extension)

b) Hypothalamic syndrome (superior tumor extension)

c) Diplopia (lateral tumor extension)

d) Parkinson-like movements (lateral tumor extension)

e) CSF rhinorrhea (inferior tumor extension)

4) Which of the following hyperfunctioning pituitary tumors is the most common?

a) Prolactinomas

Page 51: Endocrine Pre-test self test

Endocrine 05May2009

DO NOT DISTRIBUTE - 51 -

b) GH tumors resulting in acromegaly

c) ACTH tumors resulting in Cushing disease

d) TSH tumors resulting in hyperthyroidism

5) Which of the following treatments is first line therapy for GH, ACTH, and TSH

pituitary tumors?

a) Dopamine agonist

b) Surgical excision

c) Irradiation

d) Medical therapy

e) Supportive only

6) Which of the following tumors is NOT treated via trans-sphenoidal surgical excision?

a) GH tumor

b) TSH tumor

c) Prolactinoma

d) ACTH tumor

7) Which of the following may be used for prolactinomas?

a) Fenoldopam

b) Bromocryptine and cabergoline

c) Metoclopramide, droperidol, and domperidone

d) Clozapine, risperidone, and olanzapine

e) Amoxapine

8) What is the mechanism of action for drugs used against prolactinomas?

a) D1 agonists

b) D1 antagonists

c) D2 agonists

d) D2 antagonists

9) Galactorrhea and oligomenorrhea are typical in women with what type of tumor?

a) GH tumor

b) TSH tumor

c) Prolactinoma

d) ACTH tumor

10) Which of the following conditions would NOT cause hyperprolactinemia?

a) Pregnancy

b) Postpartum state

c) Stress (surgery, seizure)

d) Pituitary stalk disruption

e) Sheehan syndrome

11) Which of the following medications would NOT cause hyperprolactinemia?

a) Marijuana

b) Opiates

c) Antidepressants

d) Cimetidine

e) Verapamil

f) Cabergoline

g) Neuroleptic agents

12) Failure of what organ systems may lead to hyperprolactinemia?

Page 52: Endocrine Pre-test self test

Endocrine 05May2009

DO NOT DISTRIBUTE - 52 -

a) Renal and hepatic

b) Hepatic and splenic

c) Splenic and GI tract

d) GI tract and neurologic

e) Neurologic and renal

13) Which of the following is NOT an indication for medical treatment of a

prolactinoma?

a) Headaches

b) Infertility management

c) Hypogonadism

d) Galactorrhea

Endocrine #18 – Clinical: GH Tumors & Acromegaly

1) Which of the following is NOT a sign of acromegaly (e.g. GH-producing pituitary

tumor) in adults?

a) Prominent supraorbital ridges

b) Macroglossia (large tongue)

c) Prognathism (protruding jaw)

d) Increased hand/foot size

e) Hypertrophy of gonads

2) Which of the following is NOT associated with acromegaly?

a) Carpal tunnel syndrome symptoms

b) Excessive sweating

c) Polyuria and polydipsia

d) Increased skin oiliness

e) Headache

3) What is the best screening test for acromegaly?

a) GH serum concentration

b) IGF-1 serum concentration

c) Foot-to-femur bone length ratio

d) Jaw bone biopsy

e) Serum amylase and lipase

4) Why would a random serum level of GH be unhelpful in the diagnosis of acromegaly?

a) GH levels are always low

b) GH levels are always normal

c) GH levels are always high

d) GH levels fluctuate (pulsatile)

e) GH is not involved in acromegaly

5) What is the treatment of choice for growth hormone secreting tumors?

a) Dopamine agonist

b) Surgical excision

c) Irradiation

d) Chemotherapy

6) Pituitary apoplexy is a clinical syndrome produced by sudden hemorrhage or infarction

of the pituitary gland, often caused by a growing adenoma. Which of the following is the

most commonly seen symptom of pituitary apoplexy?

Page 53: Endocrine Pre-test self test

Endocrine 05May2009

DO NOT DISTRIBUTE - 53 -

a) Headache

b) Visual acuity defects

c) Vomiting

d) Carotid siphon

e) Horner syndrome

Endocrine #19 – Clinical: Diabetes Insipidus

1) What drugs antagonize the effects of ADH on the renal tubules, and thus could cause

nephrogenic diabetes insipidus?

a) Bromocryptine and cabergoline

b) Hydrochlorothiazide and furosemide

c) Cimetidine and verapamil

d) Lithium and demeclocycline

e) Acetaminophen and isoniazid

2) What electrolyte abnormalities can cause diabetes insipidus?

a) Hypercalcemia and hyperkalemia

b) Hypercalcemia and hypokalemia

c) Hypocalcemia and hyperkalemia

d) Hypocalcemia and hypokalemia

3) Damage to what organ would cause central diabetes insipidus?

a) Kidneys

b) Hypothalamus

c) Thyroid

d) Parathyroid

e) Pituitary

4) What are the typical presenting signs of diabetes insipidus?

a) Hyperglycemia and polyuria

b) Periorbital ecchymosis and blurred vision

c) Polyuria and polydipsia

d) Oliguria and hypoglycemia

e) Weight gain and malaise

5) Which of the following is most suggestive of psychogenic polydipsia, not diabetes

insipidus?

a) Nocturia

b) Constant symptoms

c) 24-hour urine output > 18L

d) Plasma osmolarity > 295mOsm/kg

e) Plasma osmolarity < 280mOsm/kg after a water deprivation test

6) To confirm central diabetes insipidus, post-injection (desmopressin) urine osmolarity

should be what percentage of pre-injection osmolarity?

a) < 50%

b) < 75%

c) 100% (equal)

d) > 125%

e) > 150%

Page 54: Endocrine Pre-test self test

Endocrine 05May2009

DO NOT DISTRIBUTE - 54 -

7) The drug of choice for central diabetes insipidus is desmopressin (DDAVP). What is

this drug’s mechanism of action?

a) Mimics vasopressin and increases kidney water reabsorption

b) Blocks vasopressin and increases kidney water reabsorption

c) Mimics vasopressin and increases kidney salt excretion

d) Blocks vasopressin and increases kidney salt excretion

Endocrine #20 – Clinical: Syndrome of Inappropriate ADH Secretion

1) What is the stimulus for ADH secretion in SIADH?

a) Hypovolemia

b) Hypotension

c) Hyper-osmolarity

d) None of the above

2) Which of the following is NOT a cause of SIADH?

a) Carbamazepine

b) Small cell lung carcinoma

c) Chloramphenicol

d) Cyclophosphamide

e) Pain or nausea

3) SIADH can be diagnosed via:

a) Inappropriately concentrated urine

b) Hyperosmolality

c) Hypernatremia

d) Low serum ADH

4) Extracellular volume is expended in SIADH, however, what diagnostic sign is absent?

a) Polyuria

b) Edema

c) Polydipsia

d) Headache

e) Sweating

5) Which of the following would NOT be seen in hyponatremia?

a) Lethargy

b) Nausea

c) Diarrhea

d) Confusion

6) What is the drug used for the treatment of chronic SIADH?

a) Demeclocycline

b) Desmopressin

c) Lithium

d) Bromocryptine

e) Hypotonic saline

7) In SIADH patents with acute neurologic sequelae, hypertonic saline (3%) may be

administered via IV in a dose of 200-300mL over 3-4 hours. If a rate of 0.5mEq/h is

exceeded, which of the following may occur

a) Retinal detachment

b) Acute disseminated encephalomyelitis

Page 55: Endocrine Pre-test self test

Endocrine 05May2009

DO NOT DISTRIBUTE - 55 -

c) Subarachnoid hemorrhage

d) Central pontine myelinolysis

e) Acute necrotizing hemorrhagic encephalitis

Endocrine #21 – Clinical: Disorders of the Thyroid Gland

1) What are the expected TSH levels for primary thyroid disorders?

a) Increased in both hyperthyroidism and hypothyroidism

b) Decreased in both hyperthyroidism and hypothyroidism

c) Increased in hyperthyroidism and decreased hypothyroidism

d) Decreased in hyperthyroidism and increased hypothyroidism

2) TSH receptor-stimulating immunoglobulin (TSI) is the marker for which of the

following disorders?

a) Subacute (DeQuervain) thyroiditis

b) Primary hyperthyroidism

c) Graves disease

d) Lymphocytic (Hashimoto) thyroiditis

e) Primary hypothyroidism

3) Antithyroglobulin and antimicrosomal antibodies (antiperoxidase) are seen in which of

the following?

a) Subacute (DeQuervain) thyroiditis

b) Primary hyperthyroidism

c) Graves disease

d) Lymphocytic (Hashimoto) thyroiditis

e) Primary hypothyroidism

Endocrine #22 – Clinical: Hyperthyroidism

1) What is the most common cause of hyperthyroidism?

a) Subacute (DeQuervain) thyroiditis

b) Exogenous hyperthyroidism

c) Graves disease

d) Lymphocytic (Hashimoto) thyroiditis

e) Toxic nodular goiter

2) A young female presents to the primary care clinic with nervousness. She eventually

admits to taking “several pills”, including one of her mother’s medications, to lose

weight. Which of the following is most likely?

a) Subacute (DeQuervain) thyroiditis

b) Exogenous hyperthyroidism

c) Graves disease

d) Lymphocytic (Hashimoto) thyroiditis

e) Toxic nodular goiter

3) Which of the following is NOT associated with hyperthyroidism?

a) Cold intolerance

b) Increased sweating

c) Menstrual irregularities

d) Palpitations and weight loss

e) Diarrhea and irritability

Page 56: Endocrine Pre-test self test

Endocrine 05May2009

DO NOT DISTRIBUTE - 56 -

4) Which of the following describes the biochemical diagnosis of hyperthyroidism?

a) Increased TSH and decreased FT4

b) Increased TSH and increased FT4

c) Decreased TSH and decreased FT4

d) Decreased TSH and increased FT4

5) A patient is found to dermopathy (pretibial myxedema), ophthalmopathy

(exophthalmos), and hyperthyroidism. Which of the following is most likely?

a) Subacute (DeQuervain) thyroiditis

b) Exogenous hyperthyroidism

c) Graves disease

d) Lymphocytic (Hashimoto) thyroiditis

e) Toxic nodular goiter

6) What is the cause of hyperthyroidism in Graves disease?

a) Antithyroglobulin antibodies

b) Antimicrosomal antibodies

c) T4 receptor-stimulating immunoglobulins

d) TSH receptor-inhibiting immunoglobulins

e) TSH receptor-stimulating immunoglobulins

7) What pattern is seen in postpartum thyroiditis (painless lymphocytic thyroiditis)?

a) Hyperthyroid phase then recovering to normal

b) Hypothyroid phase then recovering to normal

c) Hyperthyroid phase, then hypothyroid phase, then recovering to normal

d) Hypothyroid phase, then hyperthyroid phase, then recovering to normal

8) A patient with a history of an upper respiratory tract infections (URTI) presents with

complains of dysphagia (painful swallowing). Anterior neck palpation is painful and

tender. Corticosteroid therapy is started and shows dramatic response. Which of the

following is most likely?

a) Subacute (DeQuervain) thyroiditis

b) Exogenous hyperthyroidism

c) Graves disease

d) Lymphocytic (Hashimoto) thyroiditis

e) Toxic nodular goiter

9) A middle-aged woman presents with a solitary 4cm thyroid nodule that is easy to

palpate. A radioisotope scan demonstrates intense uptake in the nodule (“hot nodule”)

with suppressed uptake in the rest of the gland. What is this patient at risk for

developing?

a) Subacute (DeQuervain) thyroiditis

b) Exogenous hyperthyroidism

c) Graves disease

d) Lymphocytic (Hashimoto) thyroiditis

e) Toxic nodular goiter

10) Which of the following would be seen in a patient with factitial thyrotoxicosis?

a) Palpable goiter without suppressed RAIU

b) Palpable goiter with suppressed RAIU

c) No palpable goiter without suppressed RAIU

d) No palpable goiter with suppressed RAIU

Page 57: Endocrine Pre-test self test

Endocrine 05May2009

DO NOT DISTRIBUTE - 57 -

11) What is the mechanism of action for thionamides (methimazole and propylthiouracil)

used in the treatment of hyperthyroidism?

a) Prevents the release of thyroid hormone

b) Blocks the synthesis of thyroid hormone

c) Prevents the release of TSH

d) Blocks the action of TSH

e) Ablates the thyroid gland

12) What is the role of radioactive iodine in the treatment of Graves disease or

multinodular toxic goiter?

a) Prevents the release of thyroid hormone

b) Blocks the synthesis of thyroid hormone

c) Prevents the release of TSH

d) Blocks the action of TSH

e) Ablates the thyroid gland

13) What category of drugs is used to control the adrenergic manifestations of

hyperthyroidism?

a) Sodium-channel blockers, Ia (procainamide)

b) Sodium-channel blockers, Ib (lidocaine)

c) Beta-blockers (propranolol)

d) Potassium-channel blockers (amiodarone)

e) Calcium-channel blockers (verapamil)

14) A hospitalized patient with hyperthyroidism develops delirium, fever, tachycardia,

hypotension, and vomiting (thyroid storm). The patient is given sodium iodide; what is

the mechanism of action for this drug?

a) Prevents the release of thyroid hormone

b) Blocks the synthesis of thyroid hormone

c) Prevents the release of TSH

d) Blocks the action of TSH

e) Ablates the thyroid gland

Endocrine #23 – Clinical: Hypothyroidism

1) What is the most common cause of hypothyroidism worldwide?

a) Subacute (DeQuervain) thyroiditis

b) Exogenous hyperthyroidism

c) Graves disease

d) Lymphocytic (Hashimoto) thyroiditis

e) Toxic nodular goiter

2) Which of the following is NOT a symptom associated with hypothyroidism?

a) Ankle swelling

b) Dry skin and hair

c) Fatigue and achiness

d) Cold intolerance

e) Rapid weight loss

3) What electrolyte abnormality may be present in hypothyroidism?

a) Hyponatremia

b) Hypernatremia

Page 58: Endocrine Pre-test self test

Endocrine 05May2009

DO NOT DISTRIBUTE - 58 -

c) Hypokalemia

d) Hyperkalemia

4) What biochemical test results can diagnose central hypothyroidism?

a) Increased TSH and decreased FT4

b) Increased TSH and increased FT4

c) Decreased TSH and decreased FT4

d) Decreased TSH and increased FT4

5) What is the therapy for hypothyroidism?

a) Desmopressin (DDAVP)

b) Fludrocortisone (Florinef)

c) Levothyroxine (Synthroid)

d) Methimazole (Tapazole)

e) Octreotide (Sandostatin)

6) Which of the following describes subclinical hypothyroidism?

a) Decreased TSH levels in euthyroid patients with normal FT4 levels

b) Increased TSH levels in euthyroid patients with normal FT4 levels

c) Decreased TSH levels in hypothyroid patients with normal FT4 levels

d) Increased TSH levels in hypothyroid patients with normal FT4 levels

7) Which of the following would NOT be seen in a hypothyroidism patient who presents

with myxedema coma?

a) Hypotension

b) Hypothermia

c) Hyponatremia

d) Hypoventilation

e) Hyperglycemia

f) Seizures

Endocrine #24 – Clinical: Thyroid Nodules

1) What is the primary concern when discovering a thyroid nodule?

a) If hypothyroidism will occur

b) If hyperthyroidism will occur

c) If there is another underlying cause

d) If the tumor is benign or malignant

e) If SIADH and hyponatremia is present

2) What percentage of palpable thyroid nodules are benign?

a) 5%

b) 25%

c) 50%

d) 75%

e) 95%

3) Following TSH measurement, which of the following procedures is used to diagnose

the cause of a thyroid nodule?

a) Surgical resection

b) Radionuclide iodine test

c) Fine needle aspiration

d) Neck CT (or MRI)

Page 59: Endocrine Pre-test self test

Endocrine 05May2009

DO NOT DISTRIBUTE - 59 -

Endocrine #25 – Clinical: Thyroid Cancer & Miscellaneous Thyroid Disorders

1) What is the most common type of thyroid cancer?

a) Papillary carcinoma

b) Follicular carcinoma

c) Medullary carcinoma

d) Anaplastic carcinoma

2) Which of the following carries an extremely poor prognosis?

a) Papillary carcinoma

b) Follicular carcinoma

c) Medullary carcinoma

d) Anaplastic carcinoma

e) Parathyroid adenoma

3) Patients who require hospitalization for systemic illness, psychiatric disorders, or

trauma may have thyroid function test abnormalities (sick euthyroid syndrome).

Although these resolve with the recovery from the associated illness, what pattern is

typically seen?

a) Increased TSH levels throughout hospitalization

b) Decreased TSH levels throughout hospitalization

c) Increased TSH levels during acute illness and decreased TSH during recovery

d) Decreased TSH levels during acute illness and increased TSH during recovery

4) What antiarrhythmic drug can cause thyroid disease?

a) Procainamide

b) Lidocaine

c) Propranolol

d) Amiodarone

e) Verapamil

5) What psychiatric drug can cause hypothyroidism?

a) Haloperidol

b) Lithium

c) Olanzapine

d) Fluoxetine

e) Risperidone

Endocrine #26 – Clinical: Disorders of Calcium & Bone Metabolism

1) What is the most common cause of hypercalcemia in ambulatory patients?

a) Subacute (DeQuervain) thyroiditis

b) Primary hyperthyroidism

c) Graves disease

d) Lymphocytic (Hashimoto) thyroiditis

e) Primary hypothyroidism

2) What is the most common cause of primary hyperparathyroidism?

a) Parathyroid adenoma

b) Medullary carcinoma

c) Follicular carcinoma

d) Anaplastic carcinoma

Page 60: Endocrine Pre-test self test

Endocrine 05May2009

DO NOT DISTRIBUTE - 60 -

e) Papillary carcinoma

3) How are most patients with primary hyperparathyroidism diagnosed?

a) Finding an abnormal neck mass

b) Complaints of polyuria

c) Complaints of polydipsia

d) Finding a kidney stone

e) Routine lab screening

4) Which of the following is NOT a symptom of hypercalcemia?

a) Polyuria

b) Polydipsia

c) Nervousness

d) Osteodynia

e) Myopathy

5) What is the common complication of hypercalciuria?

a) Osteoporosis

b) Osteopenia

c) Phosphate deficiency

d) Nephrolithiasis

e) Urinary tract infection

6) Serum levels of PTH in primary hyperparathyroidism are usually:

a) Increased

b) Normal

c) Decreased

d) Rapidly fluctuating

7) What is the treatment of choice for primary hyperparathyroidism?

a) Dopamine agonist

b) Parathyroidectomy

c) Thyroidectomy

d) Chemotherapy

e) Irradiation

8) Which of the following is commonly seen in the early post-operative period after a

parathyroidectomy?

a) Moderate escalating hypocalcemia

b) Reversible mild asymptomatic hypocalcemia

c) Severe symptomatic hypocalcemia

d) Irreversible mild symptomatic hypercalcemia

e) Sever asymptomatic hypercalcemia

9) What is the most common cause of hypercalcemia in hospitalized patients?

a) Destructive effect of skeletal metastases

b) Paraneoplastic effect of a discrete neoplasm

c) Complete loss of thyroid function

d) The underlying cause is unknown

e) A & B

10) Serum PTH levels in hypercalcemia due to malignancy are usually:

a) Increased

b) Normal

Page 61: Endocrine Pre-test self test

Endocrine 05May2009

DO NOT DISTRIBUTE - 61 -

c) Decreased

d) Rapidly fluctuating

11) Which of the following would NOT cause hypercalcemia?

a) Vitamin D intoxication

b) Steatorrhea

c) Hyperthyroidism

d) Sarcoidosis

12) Which of the following is the treatment of choice for hypercalcemia of malignancy?

a) Glucocorticoid therapy

b) Complete resection of tumor

c) Aggressive rehydration with volume expansion (loop diuretics)

d) Pamidronate (bisphosphonate)

e) Calcitonin therapy

Endocrine #27 – Clinical: Hypoparathyroidism

1) What type of surgical intervention is most likely to damage the parathyroid glands?

a) Simple neck dissection

b) Thyroidectomy

c) Cricothyrotomy

d) Tracheostomy

e) Trans-sphenoidal surgical excision

2) What electrolyte abnormality impairs secretion of PTH?

a) Hypocalcemia

b) Hypercalcemia

c) Hyponatremia

d) Hypernatremia

e) Hypomagnesemia

3) A patient presents with short stature, round face, and short metacarpals and

metatarsals. Head CT shows calcification of the basal ganglia. Labs show increased PTH.

Further testing finds end-organ resistance to PTH (Gs subunit receptor defect). Which of

the following is most likely?

a) Hypothyroidism

b) Hyperthyroidism

c) Hypoparathyroidism

d) Hyperparathyroidism

e) Pseudohypoparathyroidism

4) What ECG abnormality is seen with hypocalcemia?

a) Peaked T waves

b) Flattened T waves

c) Shortened QT interval

d) Prolonged QT interval

e) Slurred QRS complex with a delta wave

5) For each 1gm/dL point drop in albumin, what occurs to calcium levels?

a) Decrease 0.8mg/dL

b) Decrease 1mg/dL

c) Increase 1mg/dL

Page 62: Endocrine Pre-test self test

Endocrine 05May2009

DO NOT DISTRIBUTE - 62 -

d) Increase 0.8mg/dL

e) Increase 0.6mg/dL

6) Which of the following is diagnostic of hypoparathyroidism in hypocalcemic patients?

a) High serum PTH

b) Low serum PTH

c) High serum albumin

d) Low serum albumin

e) High urine albumin

7) Which of the following should be given to a patient with severe acute hypocalcemia to

prevent tetany, stridor, or convulsions?

a) Amiodarone

b) Raloxifene

c) Calcitonin

d) Oral calcium

e) IV calcium

8) Osteoporosis is defined as bone density that is how many standard deviations below

mean (height- and sex-match control population)?

a) 1.0

b) 1.5

c) 2.0

d) 2.5

e) 3.0

9) A patient who has a bone density value between 1.0 to 2.5 standard deviations below

mean would be categorized as:

a) Osteopetrosis

b) Osteoporosis

c) Osteopenia

d) Osteodynia

e) Ostealgia

10) What are the most common types of osteoporosis?

a) Postmenopausal and pregnant

b) Pregnant and senile

c) Running and lifting related

d) Hypoparathyroidism and pseudohypoparathyroidism

e) Senile and postmenopausal

11) Which of the following endocrine disorders would NOT cause osteoporosis?

a) Hypogonadism

b) Hyperparathyroidism

c) Hypothyroidism

d) Hypercortisolism

12) Along with vertebral and distal radius (Colles) fractures, where do osteoporotic

fractures often occur?

a) Humerus

b) Pelvis

c) Proximal fibula

d) Distal tibia

Page 63: Endocrine Pre-test self test

Endocrine 05May2009

DO NOT DISTRIBUTE - 63 -

e) Phalanges

13) Which of the following serum levels is abnormal in patients with osteoporosis?

a) Calcium

b) Phosphate

c) Alkaline phosphatase

d) None of the above

14) What test can be used to assess bone mineral density of the lumbar vertebrae?

a) Dual energy x-ray absorptiometry (DEXA scan)

b) Computer tomography (CT scan)

c) Magnetic resonance imaging (MRI scan)

d) Bone scan

e) Single photon emission computed tomography (SPECT scan)

15) Which of the following is NOT used in the treatment of osteoporosis?

a) Vitamin D

b) Alendronate (Fosemax)

c) Estrogen replacement therapy

d) IV calcium

e) Calcitonin

16) Osteomalacia may be vitamin D deficiency caused by any of the following EXCEPT:

a) Poor intake (chronic alcoholism)

b) Malabsorption (celiac disease)

c) Hyperparathyroidism

d) Decreased exposure to the sun

17) Which of the following is NOT typically seen in osteomalacia?

a) Waddling gait

b) Decreased serum alkaline phosphatase

c) Muscle weakness

d) Diffuse bone pain and tenderness

18) Which of the following would NOT be an effective therapy for osteomalacia?

a) Vitamin D supplements

b) Treat the underlying cause

c) Daily sun exposure or use of tanning facilities

d) Adequate calcium and phosphate at areas of mineralization

19) Which of the following is characteristic of Paget disease?

a) Increased rate of bone resorption

b) Decreased rate of bone resorption

c) Increased rate of bone formation

d) Decreased rate of bone formation

20) Which of the following blood tests is elevated in Paget disease?

a) Calcium

b) Phosphate

c) Alkaline phosphatase

d) Serum TSH

e) Serum FT4

21) Which of the following bones is the LEAST likely to be affected by Paget disease?

a) Skull

Page 64: Endocrine Pre-test self test

Endocrine 05May2009

DO NOT DISTRIBUTE - 64 -

b) Sacrum

c) Femur

d) Radius

e) Pelvis

22) Which of the following tests that is useful in Paget disease is the most sensitive?

a) Radiograph

b) Bone scan

c) Serum alkaline phosphatase

d) Serum calcium and phosphate

e) Serum 25-hydroxy Vitamin D

23) Which of the following types of drugs is most useful for Paget disease?

a) Estrogens

b) Vitamin D

c) Calcitonin

d) Corticosteroids

e) Bisphosphonates

Endocrine #28 – Clinical: Disorders of the Adrenal Glands

1) What is the most common cause of primary adrenal failure?

a) Autoimmune adrenalitis (Addison disease)

b) Destruction by granulomatous process

c) Bilateral adrenal hemorrhage (Waterhouse-Friderichsen syndrome)

d) Drug-induced (aminoglutethiamine, ketoconazole)

e) Congenital adrenal enzyme deficiency

2) Which of the following causes of primary adrenal failure is associated with

tuberculosis (TB)?

a) Autoimmune adrenalitis (Addison disease)

b) Destruction by granulomatous process

c) Bilateral adrenal hemorrhage (Waterhouse-Friderichsen syndrome)

d) Drug-induced (aminoglutethiamine, ketoconazole)

e) Congenital adrenal enzyme deficiency

3) Which of the following signs or symptoms would NOT be seen in adrenal failure?

a) Muscle weakness

b) Weight gain

c) Nausea, vomiting, diarrhea

d) Lymphocytosis

e) Eosinophilia

4) What is the major electrolyte abnormality seen in adrenal failure?

a) Hyponatremia with hyperkalemia

b) Hyponatremia with hypokalemia

c) Hypernatremia with hyperkalemia

d) Hypernatremia with hypokalemia

e) Hyperchloremic acidosis

5) A hospitalized patient is undergoing cortisol replacement after suffering bilateral

adrenal hemorrhage. The patient develops pneumonia and complains of abdominal pain.

Shortly after, the patient develops a fever, azotemia, hypercalcemia, eosinophilia, and

Page 65: Endocrine Pre-test self test

Endocrine 05May2009

DO NOT DISTRIBUTE - 65 -

hypotension that is out of proportion to the severity of their current illness. After finding

the patient dehydrated, it is discovered that cortisol therapy was missed. Which of the

following is most likely?

a) Central pontine myelinolysis

b) Subarachnoid hemorrhage

c) Syndrome of inappropriate ADH

d) Lacunar infarction

e) Adrenal crisis

6.1) What is the therapy for secondary adrenal failure?

a) Glucocorticoid replacement only

b) Mineralocorticoid replacement only

c) Glucocorticoid and mineralocorticoid replacement

6.2) What is the therapy for primary adrenal failure?

a) Glucocorticoid replacement only

b) Mineralocorticoid replacement only

c) Glucocorticoid and mineralocorticoid replacement

7) Which of the following may be given as an IM injection for acute illness if a patient

cannot tolerate oral glucocorticoids due to nausea and vomiting?

a) Prednisone

b) Hydrocortisone

c) Dexamethasone

d) Fludrocortisone

8) Which of the following is a drug used for mineralocorticoid replacement?

a) Prednisone

b) Hydrocortisone

c) Dexamethasone

d) Fludrocortisone

Endocrine #29 – Cushing Syndrome

1) Cushing syndrome is defined as overproduction of what hormone?

a) TSH

b) PRL

c) ACTH

d) LH & FSH

e) GH

2) Which of the following defines Cushing disease, which can cause Cushing syndrome?

a) Adrenal carcinoma

b) Pituitary adenoma

c) Oat cell lung cancer

d) Prolonged use of exogenous steroids

3) Which cause of Cushing syndrome is due to ectopic ACTH production?

a) Adrenal carcinoma

b) Pituitary adenoma

c) Oat cell lung cancer

d) Prolonged use of exogenous steroids

4) Which of the following signs or symptoms is NOT consistent with Cushing syndrome?

Page 66: Endocrine Pre-test self test

Endocrine 05May2009

DO NOT DISTRIBUTE - 66 -

a) Central obesity

b) Supraclavicular fat pads

c) DM and HTN

d) Mineralocorticoid deficiency

e) Wide purple striae

5) Which of the following is NOT a sign or symptoms of adrenal androgen excess?

a) Acne

b) Hirsutism

c) Hypogonadism

d) Menstrual irregularities

6) What is the best screening test for Cushing syndrome?

a) Blood cortisol level

b) Blood estradiol level

c) Blood testosterone level

d) 24-hour urine free cortisol

e) Dexamethasone-CRH test

7) What is the best confirmatory test for the diagnosis of Cushing syndrome?

a) Blood cortisol level

b) Blood estradiol level

c) Abdominal CT scan

d) 24-hour urine free cortisol

e) Dexamethasone-CRH test

8.1) Which of the following would suggest ectopic ACTH secretion?

a) Suppressed ACTH (<5pg/mL)

b) Normal ACTH (10-80pg/mL)

c) Modestly increased ACTH (<200pg/mL)

d) Extremely increased ACTH (>200pg/mL)

8.2) What test should be performed if adrenal autonomy (ACTH-independent) Cushing

syndrome is found?

a) Blood cortisol level

b) Blood estradiol level

c) Abdominal CT scan

d) 24-hour urine free cortisol

e) Dexamethasone-CRH test

9.1) Which of the following Cushing syndrome cases would require resection and

adjuvant treatment with inhibitors of steroidogenesis (ketoconazole)?

a) Adrenal carcinoma

b) Failed trans-sphenoidal surgery

c) Adrenal adenoma

d) ACTH-dependent Cushing syndrome

e) Post bilateral adrenalectomy

9.2) What is the treatment of choice for ACTH-dependent Cushing syndrome or ACTH-

independent syndrome due to adrenal adenoma?

a) Bilateral adrenalectomy

b) Resection of the tumor

c) Resection with ketoconazole therapy

Page 67: Endocrine Pre-test self test

Endocrine 05May2009

DO NOT DISTRIBUTE - 67 -

d) Life long replacement of glucocorticoids

e) Life long replacement of mineralocorticoids

Endocrine #30 – Clinical: Primary Aldosteronism

1) Primary aldosteronism results from autonomous (renin-independent) aldosterone

production from which area?

a) Adrenal cortex, zona glomerulosa

b) Adrenal cortex, zona fasciculata

c) Adrenal cortex, zona reticularis

d) Adrenal medulla

e) Extra-adrenal

2) Which of the following is an uncommon (rare) cause of primary aldosteronism?

a) Aldosteronoma

b) Idiopathic bilateral hyperplasia

c) Familial glucocorticoid-remediable aldosteronism

d) Adrenocortical carcinoma

3) Which of the following clinical features are defining for primary aldosteronism?

a) Hypotension and hyperkalemia

b) Hypotension and hypokalemia

c) Hypertension and hyperkalemia

d) Hypertension and hypokalemia

4) What is the screening test used for primary aldosteronism?

a) Plasma aldosterone (PA)

b) Plasma renin activity (PRA)

c) PA/PRA ratio > 20 with increased PA

d) PA/PRA ratio < 20 with increased PA

e) PRA/PA ratio > 20 with decreased PA

Endocrine #31 – Clinical: Pheochromocytoma & Adrenal Incidentaloma

1) Pheochromocytomas can secrete catecholamines continuously or episodically and arise

mainly in chromaffin cells of neural crest origin in which of the following locations?

a) Adrenal cortex, zona glomerulosa

b) Adrenal cortex, zona fasciculata

c) Adrenal cortex, zona reticularis

d) Adrenal medulla

e) Extra-adrenal

2) What is the major symptom associated with pheochromocytoma?

a) Headache

b) Palpitations

c) Sweating

d) Heat intolerance

e) Paroxysmal hypertension

3) What is the urine test used to diagnose pheochromocytoma?

a) Vanillyl-mandelic acid (VMA)

b) Free catecholamines and metanephrines

c) 24-hour urine free cortisol

Page 68: Endocrine Pre-test self test

Endocrine 05May2009

DO NOT DISTRIBUTE - 68 -

d) Urobilinogen and hCG

e) Free ketone bodies

4) What tests are used to localize a pheochromocytoma?

a) CT scan of the abdomen

b) MRI scan of the abdomen

c) CT scan with 123I-MIBG

d) MRI scan with 123I-MIBG

e) Any of the above

5) Which of the following therapies is used for pheochromocytoma?

a) Preoperative medical treatment

b) Alpha-adrenergic blockade (phenoxybenzamine)

c) Beta-blockage to control reflex tachycardia

d) Complete resection of tumor

e) All of the above

6) Which of the following is NOT true of an adrenal incidentaloma?

a) Can be an adenoma

b) Can be a carcinoma

c) Can be metastatic

d) Most are functioning (secrete cortisol, aldosterone, catecholamines)

e) Most often a small (1-6cm) adrenal mass

7) What questions should be asked when addressing an incidentaloma?

a) Is the lesion benign?

b) Is the lesion malignant?

c) Is the lesion nonfunctioning?

d) Is the lesion hyperfunctioning?

e) All of the above

Endocrine #32 – Clinical: Male Testis & Hypogonadism

1) Which of the following is NOT seen in adult males with hypogonadism?

a) Decreased ejaculate volume

b) Increased sexual stamina

c) Decreased sexual hair growth

d) Hot flashes and gynecomastia

e) Decreased libido and potency

2) How does hypogonadism present in adolescence?

a) Delayed puberty and growth

b) Increased sexual urges and erections

c) Embarrassment due to female hair pattern and gynecomastia

d) Early puberty and inability to become aroused

e) Paroxysmal nocturia and eunuchoid habitus

3) A male is found to have small firm testis, suggesting Klinefelter syndrome. Which of

the following further describes this condition?

a) Hypogonadotropic hypogonadism, primary

b) Hypergonadotropic hypogonadism, primary

c) Hypogonadotropic hypogonadism, secondary

d) Hypergonadotropic hypogonadism, secondary

Page 69: Endocrine Pre-test self test

Endocrine 05May2009

DO NOT DISTRIBUTE - 69 -

4) Which of the following would suggest primary hypogonadism?

a) Low serum testosterone and low gonadotropin concentrations

b) Low serum testosterone and high gonadotropin concentrations

c) High serum testosterone and low gonadotropin concentrations

d) High serum testosterone and high gonadotropin concentrations

5) What chromosomal abnormality is seen in Klinefelter syndrome?

a) 45X

b) 46XX

c) 46XY

d) 47XXY

e) 47XYY

6) Kallman syndrome is characterized by hypogonadism and anosmia (loss of smell).

Which of the following further describes this condition?

a) Hypogonadotropic hypogonadism, primary

b) Hypergonadotropic hypogonadism, primary

c) Hypogonadotropic hypogonadism, secondary

d) Hypergonadotropic hypogonadism, secondary

7) Which of the following could cause gynecomastia?

a) Alcohol-related liver disease

b) Cimetidine

c) Spironolactone

d) Decreased testosterone production

e) All of the above

8) What blood tests are required for diagnosis of gynecomastia?

a) Testosterone and estradiol

b) LH and FSH

c) Beta-hCG

d) Prolactin and TSH

e) All of the above

Endocrine #33 – Clinical: Female Ovary & Amenorrhea

1) Which of the following describes secondary amenorrhea?

a) No menarche by age 16 with normal secondary sex characteristics

b) No menarche by age 14 without secondary sex characteristics

c) Previous menarche that has stopped for 3 months

d) Previous menarche that has stopped for 3 cycles

2) What is the most common cause of primary amenorrhea?

a) Pregnancy

b) Gonadal dysgenesis

c) Constitutional delay of puberty

d) Müllerian agensis

e) Ovarian failure

3) Levels of hCG should be measured for what cause of secondary amenorrhea?

a) Hypothalamic dysfunction

b) Polycystic ovarian syndrome

c) Pregnancy

Page 70: Endocrine Pre-test self test

Endocrine 05May2009

DO NOT DISTRIBUTE - 70 -

d) Pituitary disease

e) Ovarian failure

4) Which of the following is NOT associated with amenorrhea?

a) Hot flashes

b) Loss of secondary sex characteristics

c) Features of Turner syndrome

d) Galactorrhea

e) Vaginal wetness

5) What hormonal tests are required for diagnosis of secondary amenorrhea?

a) Testosterone and estradiol

b) LH and FSH

c) Beta-hCG

d) Prolactin and TSH

e) All of the above

6) What hormonal abnormalities indicate primary ovarian failure?

a) Increased FSH and increased estradiol

b) Increased FSH and decreased estradiol

c) Decreased FSH and increased estradiol

d) Decreased FSH and decreased estradiol

7) What hormonal abnormalities indicate a hypothalamic-pituitary disorder?

a) Increased FSH/LH and increased estradiol

b) Increased FSH/LH and decreased estradiol

c) Decreased FSH/LH and increased estradiol

d) Decreased FSH/LH and decreased estradiol

8) Estrogen replacement is combined with progesterone for what reason?

a) Prevent atrophic vaginitis

b) Loss of secondary sex characteristics

c) Control hot flashes

d) Prevent osteoporosis

e) Prevent the risk of endometrial cancer

Endocrine #34 – Clinical: Amenorrhea, Hirsutism, & Virilization

1) What chromosomal abnormality is seen in Turner syndrome?

a) 45X

b) 46XX

c) 46XY

d) 47XXY

e) 47XYY

2) Which of the following is NOT seen in Turner syndrome?

a) Webbed neck

b) Primary ovarian failure

c) Streak gonads

d) Prognathism

e) Shield-like chest

Page 71: Endocrine Pre-test self test

Endocrine 05May2009

DO NOT DISTRIBUTE - 71 -

3) Anorexia nervosa is characterized by a distorted perception of weight and body image

that leads to poor nutrition and often effects women younger than 25. Which of the

following is NOT a sign or symptoms typically seen in this disorder?

a) Amenorrhea

b) Hypotension

c) Diarrhea

d) Bradycardia

e) Lanugo hair growth

f) Increased serum cortisol

4) Which of the following is defined as androgen-induced hair grown on the androgen-

sensitive areas of the female body?

a) Virilization

b) Lanugo growth

c) Hirsutism

d) Masculinization

e) Hypogonadism

5) Masculinization of secondary sex characteristic and the sex organs is defined as:

a) Virilization

b) Lanugo growth

c) Hirsutism

d) Turner syndrome

e) Hypogonadism

6) Which of the following does NOT cause hirsutism?

a) Polycystic ovarian syndrome

b) Congenital adrenal hypoplasia

c) ACTH-dependent Cushing disease

7) What is the most common cause of hyperandrogenism in females?

a) Polycystic ovarian syndrome

b) Turner syndrome

c) ACTH-dependent Cushing disease

d) Anorexia nervosa

e) Congenital adrenal hyperplasia

8) What metabolic disorders are associated with polycystic ovary syndrome?

a) Hyperthyroidism and type-2 diabetes mellitus

b) Insulin resistance and type-1 diabetes mellitus

c) Insulin resistance and type-2 diabetes mellitus

d) Glucose intolerance and type-1 diabetes mellitus

e) Glucose intolerance and type-2 diabetes mellitus

9) Autosomal recessive deficiency of one of the steroidogenic enzymes necessary for the

synthesis of corticosteroid hormones is known as:

a) Polycystic ovarian syndrome

b) Turner syndrome

c) ACTH-dependent Cushing disease

d) Anorexia nervosa

e) Congenital adrenal hyperplasia

10) What is the most common enzyme deficiency in congenital adrenal hyperplasia?

Page 72: Endocrine Pre-test self test

Endocrine 05May2009

DO NOT DISTRIBUTE - 72 -

a) 3beta-hydroxy SDH

b) 11beta-hydroxylase

c) 17alpha-hydroxylase

d) 21-hydroxylase

11) Oral contraceptives and what other drug are used in the management of idiopathic

hirsutism and polycystic ovarian syndrome?

a) Rimantadine

b) Hydrocortisone

c) Spironolactone

d) Dexamethasone

e) Danazol

Endocrine #35 – Clinical: Hyperlipidemias

1) What lipoprotein is associated with decreased risk of heart disease?

a) Increased HDL

b) Decreased HDL

c) Increased LDL

d) Increased chylomicrons

e) Increased VLDL

2) Triglyceride-rich lipoproteins (>1000mg/dL) is associated with which of the

following?

a) Acute cholecystitis

b) Alcoholic fatty liver

c) Nephrolithiasis

d) Acute pancreatitis

e) Cholelithiasis

3.1) When should diet be initiated for a patient without coronary artery disease (CAD)

and two or more risk factors?

a) 100mg/dL

b) 130mg/dL

c) 160mg/dL

d) 190mg/dL

3.2) When should drug therapy be initiated for a patient without CAD and with fewer

than two risk factors?

a) 100mg/dL

b) 130mg/dL

c) 160mg/dL

d) 190mg/dL

4) Hypothyroidism, nephrotic syndrome, and anabolic steroids cause what type of

hyperlipidemia?

a) Increased HDL

b) Decreased HDL

c) Increased LDL

d) Decreased LDL

5) What is the drug of choice for the treatment of increased concentration of LDL?

a) Niacin

Page 73: Endocrine Pre-test self test

Endocrine 05May2009

DO NOT DISTRIBUTE - 73 -

b) Fibric acid derivatives

c) Calcium-channel blockers

d) Beta-blockers

e) Statins

6) What is the drug of choice for the treatment of hypertriglyceridemia?

a) Niacin

b) Fibric acid derivatives

c) Calcium-channel blockers

d) Beta-blockers

e) Statins

Endocrine #36 – Clinical: Diabetes Mellitus

1) Diabetes mellitus is a metabolic disorder characterized by:

a) Increased fasting serum glucose and increased postprandial serum glucose

b) Increased fasting serum glucose and decreased postprandial serum glucose

c) Decreased fasting serum glucose and increased postprandial serum glucose

d) Decreased fasting serum glucose and decreased postprandial serum glucose

2) Which of the following is associated with type-1 diabetes mellitus (childhood onset)?

a) Polyuria

b) Polydipsia

c) Polyphagia

d) Ketoacidosis

e) Insulin resistance

3) Diabetes mellitus requires fasting plasma glucose levels on 2 or more occasions to be:

a) < 74mg/dL

b) < 100mg/dL

c) > 100mg/dL

d) > 126mg/dL

e) > 152mg/dL

4) What is the cornerstone therapy for type-1 diabetes mellitus?

a) Exercise to enhance glycemic response

b) Protein 10-20% of calories

c) Total fat < 30% of calories

d) Saturated fat < 10 of calories

e) Insulin replacement

5) The conclusions of the diabetes control and complications trial (DCCT) stated that

intensive therapy with tight glycemic control prevents or markedly decreased the risk of

what diabetes complication?

a) Diabetic macrovascular disease

b) Hyaline arteriosclerosis

c) Diabetic microangiopathy

d) Diabetic nephropathy

e) Diffuse mesangial sclerosis

f) Nodular glomerulosclerosis

6) What is the goal for hemoglobin A1c in optimal therapy?

a) < 7%

Page 74: Endocrine Pre-test self test

Endocrine 05May2009

DO NOT DISTRIBUTE - 74 -

b) > 7%

c) < 22%

d) > 22%

e) < 27%

7) Treatment goals for patients with type-2 diabetes include:

a) Exercise

b) Nutrition

c) Modification of hypertension

d) Modification of dyslipidemia

e) All of the above

8) Which of the following drugs used in the treatment of type-2 diabetes inhibits brush

border enzymes, which decreases digestion of ingested carbohydrates?

a) Sulfonylureas

b) Biguanides

c) Thiazolidinediones

d) Repaglinide and nateglinide

e) Alpha-glycosidase inhibitors

9) What is the mechanism of action for sulfonylureas (glipizide, glyburide, glimepiride)?

a) Increase insulin secretion (secretagogue)

b) Decrease glucose production in the liver (gluconeogenesis)

c) Increase insulin sensitivity and glucose uptake at target tissue

d) Inhibit brush border enzymes to decrease carbohydrate ingestion

10) What is the mechanism of action for biguanides (metformin)?

a) Increase insulin secretion (secretagogue)

b) Decrease glucose production in the liver (gluconeogenesis)

c) Increase insulin sensitivity and glucose uptake at target tissue

d) Inhibit brush border enzymes to decrease carbohydrate ingestion

11) What is the mechanism of action for thiazolidinediones (rosiglitazone, pioglitazone)?

a) Increase insulin secretion (secretagogue)

b) Decrease glucose production in the liver (gluconeogenesis)

c) Increase insulin sensitivity and glucose uptake at target tissue

d) Inhibit brush border enzymes to decrease carbohydrate ingestion

12) When is insulin NOT the preferred therapy in patients with type-2 diabetes?

a) During pregnancy

b) Severely ill patients

c) Patients undergoing surgical treatment

d) Prior to the use of diet and oral agents

13) What is the typical combination therapy for a patient with type-2 diabetes?

a) Sulfonylurea and rosiglitazone

b) Sulfonylurea and metformin

c) Rosiglitazone and repaglinide

d) Alpha-glycosidase inhibitor and metformin

e) Thiazolidinediones and alpha-glycosidase inhibitor

14) What organ impairment decreases insulin clearance?

a) Pancreas

b) Liver

Page 75: Endocrine Pre-test self test

Endocrine 05May2009

DO NOT DISTRIBUTE - 75 -

c) GI tract

d) Kidney

e) Muscle

15) A type-1 diabetic undergoing ketoacidosis may present with anorexia, tachycardia,

polyuria, polydipsia, abdominal pain, acetone breath, and what type of breathing pattern?

(Kussmaul)

a) Deep and rapid

b) Shallow and rapid

c) Deep and slow

d) Shallow and slow

16) Ketoacidosis involves hyperglycemia, ketonemia, and what blood gas abnormality?

a) Metabolic alkalosis

b) Metabolic acidosis

c) Respiratory alkalosis

d) Respiratory acidosis

17) All of the following electrolytes may be depleted in the body despite normal serum

levels EXCEPT:

a) Potassium

b) Phosphate

c) Calcium

d) Magnesium

18) Diabetic ketoacidosis is treated with fluids (normal saline), insulin therapy, and what

electrolyte (phosphate added if level is <1gm/dL)?

a) Sodium

b) Potassium

c) Magnesium

d) Calcium

19) Which of the following is NOT seen in hyperglycemic hyperosmolar coma?

a) Marked hyperglycemia (often >600mg/dL)

b) Hyperosmolar dehydration

c) Ketoacidosis with Kussmaul breathing

d) Mental status changes

e) Precipitated by an acute illness or infection

20) Hyperglycemic hyperosmolar coma is a complication of poorly controlled:

a) Type-1 diabetes mellitus

b) Type-2 diabetes mellitus

c) Diabetes insipidus

21) Rapid administration of which of the following could lead to cerebral edema in

patients being treated for hyperglycemic hyperosmolar non-ketonic coma?

a) Normal saline (0.9%)

b) Hypotonic saline (0.45%)

c) IV insulin

d) Subcutaneous insulin

e) Electrolyte replacement

22) What microvascular complications are seen in long-term diabetics?

a) Retinopathy

Page 76: Endocrine Pre-test self test

Endocrine 05May2009

DO NOT DISTRIBUTE - 76 -

b) Nephropathy

c) Neuropathy

d) All of the above

23) Which of the following distinguishes proliferative retinopathy (PDR) from

background diabetic retinopathy (NPDR)?

a) Microaneurysms

b) Hard exudates

c) Hemorrhages

d) Macular edema

e) New fragile vessels

24) What is the treatment used for proliferative retinopathy?

a) Panretinal photocoagulation

b) Canaloplasty

c) Vitrectomy

d) Keratomilleusis

e) Pterygium excision

25) Which of the following cutaneous infections often seen in type-2 diabetics is

considered life threatening?

a) Candidiasis

b) Furnuncle (Staph. aureus)

c) Carbuncle (Staph. aureus)

d) External otitis (Pseudomonas)

26) How does the risk of myocardial infarction compare for patients who have previously

had a myocardial infarction with patients who have diabetes?

a) Higher risk in diabetics

b) Higher risk in past MI patients

c) Same risk

27) Which of the following complications of inadequate glycemic control in pregnant

patients occurs early on in pregnancy?

a) Neonatal hypoglycemia

b) Hypocalcemia

c) Polycythemia

d) Hyperbilirubinemia

e) Congenital malformations

Endocrine #37 – Clinical: Hypoglycemia in Non-Diabetic Patients

1) Which of the following would NOT cause insulin-mediated hypoglycemia?

a) Insulinoma

b) Exogenous insulin use

c) Sulfonylurea use

d) Thiazolidinedione use

e) Autoimmune hypoglycemia

2) Failure of what organs in hospitalized patients can cause noninsulin-mediated

hypoglycemia?

a) Liver and kidneys

b) Kidneys and pancreas

Page 77: Endocrine Pre-test self test

Endocrine 05May2009

DO NOT DISTRIBUTE - 77 -

c) Pancreas and gallbladder

d) Gallbladder and spleen

e) Spleen and liver

3) Which of the following is NOT a sign of hypoglycemia?

a) Confusion and diplopia

b) Inappropriate affect

c) Bradycardia

d) Nervousness

e) Tremor and sweating

4) Diagnosis of hypoglycemia involves the Whipple triad, which includes all of the

following EXCEPT:

a) Low plasma glucose via lab testing

b) Low blood glucose via monitor

c) Presence of hypoglycemic symptoms

d) Prompt resolution of symptoms when plasma glucose is increased to normal

5) Which of the following would indicate that a patient has been injecting insulin?

a) Hypoglycemia and high C peptide

b) Hypoglycemia and low C peptide

c) Hyperglycemia and high C peptide

d) Hyperglycemia and low C peptide

6) Which of the following glucose levels would be indicative of hypoglycemia?

a) 0mg/dL

b) 45mg/dL

c) 65mg/dL

d) 85mg/dL

e) 105mg/dL

7) Plasma sulfonylurea levels should be measured in hypoglycemic patients, as

sulfonylurea use is indistinguishable from:

a) Insulinoma

b) Exogenous insulin use

c) Autoimmune hypoglycemia

d) Postprandial hypoglycemia

8) A patient demonstrates endogenous hyperinsulinemic hypoglycemia. Their plasma

concentration of insulin is > 6mU/mL and their C peptide is > 200pmol/L. Which of the

following is most likely?

a) Insulinoma

b) Exogenous insulin use

c) Autoimmune hypoglycemia

d) Postprandial hypoglycemia

9) Postprandial hypoglycemia involves symptomatic hypoglycemia how soon after a

meal?

a) 0-15 minutes

b) 15-30 minutes

c) 30-60 minutes

d) 1-5 hours

e) 6-12 hours

Page 78: Endocrine Pre-test self test

Endocrine 05May2009

DO NOT DISTRIBUTE - 78 -

10) What is the treatment for postprandial hypoglycemia in patients who are unable to eat

or drink?

a) 1mg insulin IM

b) 2mL 50% IV dextrose

c) 1mg glucose IO

d) 1mL 50% IV dextrose

e) 1mg glucagon IM

Endocrine #38 – Clinical: Multiple Endocrine Neoplasia

1) Which of the following neoplasm is NOT included in MEN-1?

a) Parathyroid

b) Thyroid

c) Endocrine pancreas

d) Anterior pituitary

2) What is the most common pituitary tumor in MEN-1?

a) GH tumor

b) TSH tumor

c) Prolactinoma

d) ACTH tumor

3) Which of the following neoplasms is NOT included in MEN-IIA?

a) Medullary carcinoma of the thyroid (MTC)

b) Pheochromocytoma

c) Primary hyperparathyroidism

d) Follicular carcinoma of the thyroid

4) Which of the following is NOT a feature of MEN-IIB?

a) MTC

b) Pheochromocytoma

c) Marfanoid habitus

d) Mucosal neuromas

e) Cushing syndrome

Endocrine #39 – Extra: Common Fractures of the Lower Limb

1) Which of the following common fractures of the knee is NOT categorized as distal

femur fracture?

a) Supracondylar

b) Intercondylar

c) Condylar

d) Tibial plateau

e) Epiphyseal

2) Damage to what nerve of the lower limb would lead to foot drop?

a) Tibial

b) Fibular

c) Peroneal

d) Sciatic

3) Immediate hemarthrosis of the knee following an injury is an important physical

finding that usually indicates an intra-articular fracture or disruption of what ligament?

Page 79: Endocrine Pre-test self test

Endocrine 05May2009

DO NOT DISTRIBUTE - 79 -

a) Anterior cruciate ligament (ACL)

b) Posterior cruciate ligament (PCL)

c) Lateral collateral ligament (LCL)

d) Medial collateral ligament (MCL)

e) Medial meniscus

4) Delayed onset of that occurs 12 to 24 hours after an injury would indicate a torn:

a) Anterior cruciate ligament (ACL)

b) Posterior cruciate ligament (PCL)

c) Lateral collateral ligament (LCL)

d) Medial collateral ligament (MCL)

e) Medial meniscus

5) What is classically the most reliable sign of underlying fracture, although it may be

less useful in children?

a) Effusion

b) Muscle spasm

c) Point tenderness

d) Stability testing

e) Range of motion testing

6) Which of the following is the most important aspect in the assessment of ligamentous

injuries that often accompany fractures of the knee?

a) Effusion

b) Muscle spasm

c) Point tenderness

d) Stability testing

e) Range of motion testing

7) An avulsion fracture of the lateral tibial plateau, also known as a Segond fracture,

predicts the presence of an ACL rupture in 75% to 100% of cases.What is the test of

choice in this case?

a) Anterior drawer test

b) Posterior drawer test

c) Lachman test

d) McMurray test

e) Apley grind test

8) Which of the following patients would NOT need a knee x-ray under the Ottawa Knee

Rules?

a) Patient older than 55

b) Bony tenderness at the head of the fibula

c) Isolated tenderness of the patella

d) Patient can hyperflex their knee well beyond 90-degrees

e) Patient cannot transfer weight for four steps

9) What radiographic view is useful in detecting tibial spine fractures or avulsion

fractures at the site of ACL or PCL attachment?

a) Sunrise (skyline) view

b) Lateral view

c) Tunnel view

d) Anterior-posterior view

Page 80: Endocrine Pre-test self test

Endocrine 05May2009

DO NOT DISTRIBUTE - 80 -

e) Oblique view

10) What view is utilized to image the patella when a patellar fracture or subluxation is

suspected?

a) Sunrise (skyline) view

b) Lateral view

c) Tunnel view

d) Anterior-posterior view

e) Oblique view

11) When examining a plain film of the knee, it is useful to assess for all of the following

EXCEPT:

a) Alignment

b) Bony cortical defects

c) Cartilage radiolucency

d) Deviation of ligamentous attachments

12) Which of the following is especially helpful to guide operative intervention or when

the diagnosis is unclear or a specific fracture classification is needed?

a) Plain film of the knee

b) MRI of the knee

c) Bone scan

d) CT of the knee

e) Vascular studies

13) Which of the following is most often used to evaluate injuries of the cartilage,

menisci, and ligaments, it can also be used to detect occult fractures or contusions to the

bone?

a) Plain film of the knee

b) MRI of the knee

c) Bone scan

d) CT of the knee

e) Vascular studies

14) Post arthrocentesis injection of morphine 1 to 5mg diluted in normal saline solution

to a total volume of 30ml can provide pain relief for:

a) 3-hours

b) 6-hours

c) 12-hours

d) 24-hours

e) 48-hours

15) Fractures of the tibial plateau are at significant risk for vascular complications,

especially damage to the popliteal artery, and may damage what nerve?

a) Tibial

b) Fibular

c) Peroneal

d) Sciatic

16) In a Segond fracture, there is a bony avulsion of the lateral tibial plateau that appears

as an oval-shaped fragment adjacent to the lateral tibial plateau (lateral capsule sign).

This is an important marker of disruption to which of the following?

a) Anterior cruciate ligament (ACL)

Page 81: Endocrine Pre-test self test

Endocrine 05May2009

DO NOT DISTRIBUTE - 81 -

b) Posterior cruciate ligament (PCL)

c) Lateral collateral ligament (LCL)

d) Medial collateral ligament (MCL)

e) Medial meniscus

17) Which of the following is NOT recommended during the first 48-hours of an acute

knee injury?

a) Rest

b) Ice

c) Compression

d) Elevation

e) Casting

18) What Salter-Harris category of epiphyseal fracture is the most common and is a

fracture through the growth plate and the metaphysis, sparing the epiphysis?

a) Type I

b) Type II

c) Type III

d) Type IV

e) Type V

James Lamberg

Page 82: Endocrine Pre-test self test

Endocrine 05May2009

DO NOT DISTRIBUTE - 82 -

AnswerKey

Endo #1

1) C

2.1) E

2.2) D

3.1) A

3.2) B

3.3) E

4.1) A

4.2) E

4.3) C

4.4) A

4.5) D

4.6) D

4.7) C

4.8) E

4.9) B

4.10) A

4.11) A

4.12) C

4.13) D

4.14) E

4.15) A

5.1) B

5.2) D

5.3) F

6.1) C

6.2) D

7.1) A

7.2) B

7.3) C

8.1) D

8.2) E

9.1) D

9.2) C

9.3) D

9.4) A

9.5) C

9.6) B

9.7) C

9.8) A

9.9) B

10.1) A

10.2) B

10.3) D

11) D

Endo #2

1) A

2) D

3) B

4) C

5) B

6) D

7) E

8) D

9) E

10) B

11) B

12) C

13) D

14) A

15) D

16) E

17) E

18) C

19) D

20) B

21) C

22) D

23) E

Endo #3

1) D

2) A

3) C

4) A

5) D

6) B

7) E

8) D

9) C

10) A

11) A

12) B

13) B

14) B

15) E

16) D

17) C

18) A

19) C

20) C

21) A

22) B

23) C

24) E

25) B

26) E

Endo #4

1) B

2) D

3) C

4) B

5) B

6) A

7) E

8) D

9) D

10) E

11) B

12) C

13) D

14) A

15) C

16) A

17) D

18) B

19) C

20) C

21) B

22) B

23) E

24) C

25) C

26) C

27) A

28) B

Endo #5

1) D

2) A

3) B

4) C

5) E

6) C

7) B

8) C

9) A

10) D

11) C

12) D

13) C

14) A

15) B

16) D

17) C

18) D

19) B

20) D

21) B

22) A

23) E

24) A

25) B

26) B

27) A

28) C

29) C

Endo #6

1) B

2) C

3) A

4) E

5) D

6) E

7) D

8) A

9) E

10) A

11) D

12) C

13) A

14) B

15) E

16) A

17) A

18) A

19) D

Endo #7

1) C

2) B

3) D

4) A

5) B

6) C

7) D

8) A

9) B

10) D

11) D

12) B

13) E

14) A

15) E

Endo #8

1) B

2) A

3) B

4) E

5) C

6) D

7) A

8) C

9) C

10) B

11) B

12) E

13) D

14) A

15) D

16) C

17) E

18) E

Endo #9

1) B

2.1) E

2.2) C

3.1) F

3.2) C

4) C

5) A

6) B

7) A

8) D

Page 83: Endocrine Pre-test self test

Endocrine 05May2009

DO NOT DISTRIBUTE - 83 -

Endo #10

1.1) B

1.2) D

1.3) B

1.4) E

2) A

3) D

4) C

5) D

6) D

Endo #11

1.1) D

1.2) A

1.3) B

2) C

3) A

4) D

5) B

6) E

7) B

8) D

Endo #12

1.1) D

1.2) C

2) C

3) A

4) B

5) C

6) D

7) E

8) C

9.1) E

9.2) A

10) B

11) D

12.1) D

12.2) B

Endo #13

1.1) B

1.2) D

2) A

3) B

4) B

5) C

6) E

7) D

8) D

9) C

10) D

11) C

12) A

13) B

14) E

15) C

Endo #14

1) E

2) A

3) C

4) D

5) B

6) B

7) D

Endo #15

1) C

2.1) C

2.2) A

2.3) B

2.4) F

2.5) C

2.6) E

2.7) B

2.8) B

3.1) A

3.2) D

3.3) B

3.4) C

3.5) E

4.1) B

4.2) C

4.3) B

5) E

6) D

7) B

8) A

9) D

10) A

11) C

12) D

13) D

14) A

15) C

16) D

17) C

18.1) C

18.2) A

18.3) E

18.4) B

19.1) D

19.2) B

19.3) D

20) C

21) D

22.1) A

22.2) D

23.1) A

23.2) B

23.3) D

23.4) A

23.5) D

24) C

25) E

26) B

27) C

28) A

29) D

30) D

31) D

32) B

33) D

34) D

35) D

36) E

37.1) C

37.2) D

37.3) C

Endo #16

1) B

2) D

3) E

4) A

5) D

6) B

7) A

8) B

9) D

10) C

Endo #17

1) A

2) D

3) D

4) A

5) B

6) C

7) B

8) C

9) C

10) E

11) F

12) A

13) A

Endo #18

1) E

2) C

3) B

4) D

5) B

6) A

Endo #19

1) D

2) B

3) E

4) A

5) C

6) E

7) A

Endo #20

1) D

2) C

3) A

4) B

5) C

6) A

7) D

Endo #21

1) D

2) C

3) D

Endo #22

1) C

2) B

3) A

4) D

5) C

6) E

7) C

8) A

9) E

10) C

11) B

12) E

13) C

14) A

Endo #23

1) D

2) E

3) A

4) C

5) C

6) B

7) E

Endo #24

1) D

2) E

3) C

Endo #25

1) A

2) D

3) D

4) D

5) B

Endo #26

1) B

2) A

3) E

Page 84: Endocrine Pre-test self test

Endocrine 05May2009

DO NOT DISTRIBUTE - 84 -

4) C

5) D

6) A

7) B

8) B

9) E

10) C

11) B

12) B

Endo #27

1) B

2) E

3) E

4) D

5) A

6) B

7) E

8) D

9) C

10) E

11) C

12) B

13) D

14) A

15) D

16) C

17) B

18) C

19) A

20) C

21) B

22) B

23) E

Endo #28

1) A

2) B

3) B

4) A

5) E

6.1) A

6.2) C

7) C

8) D

Endo #29

1) C

2) B

3) C

4) D

5) C

6) D

7) E

8.1) D

8.2) C

9.1) A

9.2) B

Endo #30

1) A

2) C

3) D

4) C

Endo #31

1) D

2) E

3) B

4) E

5) E

6) D

7) E

Endo #32

1) B

2) A

3) B

4) B

5) D

6) C

7) E

8) E

Endo #33

1) D

2) B

3) C

4) E

5) E

6) B

7) D

8) E

Endo #34

1) A

2) D

3) C

4) C

5) A

6) B

7) A

8) E

9) E

10) D

11) C

Endo #35

1) A

2) D

3.1) B

3.2) D

4) C

5) E

6) B

Endo #36

1) A

2) D

3) D

4) E

5) C

6) A

7) E

8) E

9) A

10) B

11) C

12) D

13) B

14) D

15) A

16) B

17) C

18) B

19) C

20) B

21) C

22) D

23) E

24) A

25) D

26) C

27) E

Endo #37

1) D

2) A

3) C

4) B

5) B

6) B

7) A

8) A

9) D

10) E

Endo #38

1) B

2) C

3) D

4) E

Endo #39

1) D

2) B

3) A

4) E

5) C

6) D

7) C

8) D

9) C

10) A

11) D

12) D

13) B

14) D

15) C

16) A

17) E

18) B